USABO Semifinal 2013 Answer Key

You might also like

Download as pdf or txt
Download as pdf or txt
You are on page 1of 98

USABO 2013 Semifinal Answer Key –NO OFFICIAL ANSWERS HAS BEEN PUBLISHED YET

Transmembrane region is highly hydrophobic, thus only hydrophobic amino acids will be found there. Remember
the table with the main properties of amino acids:

All amino acids mentioned in the problem are hydrophobic .Now remember that transmembrane regions are
mainly formed by alpha helices. An alpha helix is especially suited for cross-membrane proteins because all of the
amino hydrogen and carbonyl oxygen atoms of the peptide backbone can interact to form intrachain hydrogen
bonds while its aliphatic side chains can stabilize in hydrophobic environment of cell membrane. Proline is an
amino acid that destabilizes alpha helix because of its irregular geometry; its R-group bonds back to the nitrogen
of the amide group, which causes steric hindrance. In addition, the lack of a hydrogen on proline's nitrogen
prevents it from participating in hydrogen bonding. Proline is not an amino acid, but rather an imino acid.
Proline peptide bonds are found in the cis configuration ~100 times as often as those between other amino acids.
There is no H-bond donor in the proline peptide bond and the presence of
proline leads to a bend or kink in the polypeptide chain.
Therefore, the correct answer is C. Proline.

I would choose B because hydrogen bonds form between hydrogen atom


and electronegative atom (F, O, N). Thus only B has enough hydrogens:

Ribulose-1,5-bisphosphate carboxylase oxygenase (statement E is


correct), normally shortened to RuBisCO, is the most abundant
enzyme on Earth (statement A is correct; the same sentence can be
found in Campbell as well) – and arguably one of the most
important for life (at least as we know it!). It is the only net
carboxylase that we know of – that is, the only enzyme capable of
catalysing a net fixation of inorganic carbon into organic molecules
in one step. This plays a central role in the process of
photosynthesis, as it has since RuBisCO first evolved, about 3.6 billion years ago.
RuBisCO catalyses the important entry step of CO2 into the Calvin-Benson-Bassham (CBB) cycle, as part of the
light-independent reactions of photosynthesis. CO2 is reacted with ribulose-1,5-bisphosphate to produce an
unstable 6-carbon intermediate that rapidly dissociates into two molecules of 3-phosphoglycerate (3-PGA). Thus,
statement D is correct. Ultimately this gives rise to triose phosphates from which sugars can be derived. If you
take a look at the diagram on the right you will come to conclusion that Rubisco is involved in carbon fixation
step, thus statement B is correct.
Statement C is incorrect one because not carbon atoms bind to rubisco but the entire molecule of carbon dioxide.
So the answer may be C.

The central dogma looks like this:

If we have a codon made of three nucleotides and we have 4 different types of nucleic acids, then we can have 4 x
4 x 4 = 64 amino acids. Thus I would choose answer E and ignore the fact that I know that 3 codons do not specify
any amino acid but instead are stop codons (but we are not told about this in the problem).
Let’s analyse the normal expression pattern first:
• When we have low glucose and low lactose, we have very low levels of operon expression (+).
• When we have low lactose and high concentration of glucose, we have no activity of operon (-), meaning
that presence of glucose inhibits/suppresses operon activity.
• When we have high levels of lactose and low levels of glucose (repression is minimal), we have highest
activity of operon (+++).
• When we have high lactose and high glucose concentrations, lac operon is active but slightly repressed by
glucose (++).
Now let’s look at our mutant.
• When we have low glucose and low lactose, we have very high levels of operon expression (+++).
• When we have low lactose and high concentration of glucose, we have high activity of operon (++),
meaning that presence of glucose slightly inhibits/suppresses operon activity.
• When we have high levels of lactose and low levels of glucose, we have very high activity of operon (+++).
• When we have high lactose and high glucose concentrations, lac operon is active but slightly repressed by
glucose (++).
Now we can compare wild type and mutant and consider each statement.
If we lose repressor allolactose binding site, this means that allolactose no longer binds to the repressor and
cannot relieve repression, thus operon is locked in inactive state. This would mean no or very little expression of
operon, which is not the case in our problem. So we can rule out statement A.
If a mutation affects repressor’s DNA binding site, this means that the repressor is no longer able to bind DNA
and do not cause repression. This would result in constitutive activation of operon. This could explain very high
activity in mutant’s operon when we have low concentration of both glucose and lactose and increased activity
when we have high lactose and high glucose concentrations as compared to wild-type. Thus statement B is very
probable.
If a mutation occurs in operon, a repressor can no longer bind to the altered sequence, thus resulting in no
repression and constitutive activation of lac operon. This again could explain high activity of lac operon in a
mutant when we have low lactose concentration and low/high concentration of glucose. Thus we can also choose
statement C.
Lac Z encodes beta-galactosidase which cleaves lactose. This enzyme is encoded in the operon itself (also called a
structural gene). If a mutation occurs in lacZ, this would not affect operon activity but would result in altered
metabolism of sugars. Thus, D is incorrect.
LacY encodes lactose permease which is a membrane protein. Lactose permease can be classified as a symporter,
which uses the proton gradient towards the cell to transport β-galactosidase such as lactose in the same direction
into the cell. If no permease is present, we do not have a channel in membrane which would allow lactose to enter
and be converted to allolactose. Allolactose binds to repressor and relives lac operon repression. So if permease
is not present, no allolactose is present and the repressor is tightly bound to operon, inhibiting the lac operon.
This is not the case in this mutant which shows great activity of operon.

Firstly, let’s look at the expression patterns in this mutant:


• When we have low glucose and low lactose, we have low levels of operon expression (+) as in wild-type.
• When we have low lactose and high concentration of glucose, we no activity of operon (-), again as in
wild-type.
• When we have high levels of lactose and low levels of glucose, we have low activity of operon (+), smaller
than in wild-type (+++).
• When we have high lactose and high glucose concentrations, lac operon is inactive (-), totally different
from wild-type (++).

If we lose repressor allolactose binding site, this means that allolactose no longer binds to the repressor and
cannot relieve repression, thus operon is locked in inactive state. This would mean no or very little expression of
operon, which is the case in our problem. So we can choose statement A.
If a mutation affects repressor’s DNA binding site, this means that the repressor is no longer able to bind DNA
and do not cause repression. This would result in constitutive activation of operon. This could NOT explain very
low or no activity in mutant’s operon. Thus we can rule out statement B.
If a mutation occurs in operon, a repressor can no longer bind to the altered sequence, thus resulting in no
repression and constitutive activation of lac operon. This again could NOT explain low activity of lac operon in a
mutant. Thus we can also rule out statement C.
Lac Z encodes beta-galactosidase which cleaves lactose. This enzyme is encoded in the operon itself (also called a
structural gene). If a mutation occurs in lacZ, this would not affect operon activity but would result in altered
metabolism of sugars. Thus, D is incorrect.
LacY encodes lactose permease which is a membrane protein. Lactose permease can be classified as a symporter,
which uses the proton gradient towards the cell to transport β-galactosides such as lactose in the same direction
into the cell. If no permease is present, we do not have a channel in membrane which would allow lactose to enter
and be converted to allolactose. Allolactose binds to repressor and relives lac operon repression. So if permease
is not present, no allolactose is present and the repressor is tightly bound to operon, inhibiting the lac operon.
This is the case in this mutant which shows very low activity of operon. So statement E is also correct.
Remember that FADH2 is found in complex II of electron transport chain whereas NADH binds to complex I (also
known as NADH dehydrogenase).Cytochrome c, cytochrome c oxidase, cytochrome reductase, and ubiquinone are
all downstream the complex II, thus mutations in these complexes will affect both NADH and FADH2 metabolism.
Therefore, only mutation of NADH dehydrogenase which is upstream of complex II will affect metabolism of
NADH but not FADH2.The answer should be E.

One of the primers is 5’-AACTGA……TTTCAC -3’ (complementary to the sequence in problem). The other should
be complementary to the first primer: 5’- GTGAAA…….CAGTT-3’. Thus, the answer should be B.
Starch is is a carbohydrate consisting of a large number of glucose units joined by glycosidic bonds. This
polysaccharide is produced by most green plants as an energy store. Pure starch is a white, tasteless and odorless
powder that is insoluble in cold water or alcohol. It consists of two types of molecules: the linear and helical
amylose and the branched amylopectin. Depending on the plant, starch generally contains 20 to 25% amylose and
75 to 80% amylopectin by weight.
Maltose is a disaccharide made of two subunits of glucose, therefore, when starch is digested some of its glucose
subunits are not cleaved individually but are cleaved to maltose (the enzyme responsible for this is amylase). Now
note that both A and C molecules are made of glucose differing only in alpha and beta configurations. α-and β
Carbohydrates are determined by position of the OH group attached to the anomeric carbon and the CH2OH
group attached to the other carbon next to the ether.
Alpha and Beta linkages are found in disaccharides and polysaccharides. These glycosidic linkages are the bonds
between two simple sugars within a disaccharide or polysaccharide. Alpha linkages are easily digested by the
human body. Beta linkages are stronger than Alpha linkages because they are more stable. Carbohydrates with
Beta linkages are not easily digested by the human body, except for lactose because most humans have an enzyme
which breaks down this disaccharide.
These glycosidic linkages cause the anomeric carbon to be in a fixed position. Alpha glycosidic linkages cause the
anomeric carbon to be fixed in the Alpha configuration. Beta configurations cause the anomeric carbon to be fixed
in a Beta configuration.
Because starch is easily digested by humans (compare with cellulose which is made of beta-glucose), we can
conclude that starch has alpha-glucose subunits.
The answer should be C.
Remember that bacteria do not have nucleus, organelles, and cytoskeleton. Thus you can rule out A, B, C, and D.
You should choose E. Flagellin. Flagellin is a globular protein that arranges itself in a hollow cylinder to form the
filament in a bacterial flagellum. It has a mass of about 30,000 to 60,000 daltons. Flagellin is the principal
substituent of bacterial flagellum, and is present in large amounts on nearly all flagellated bacteria. The structure
of flagellin is responsible for the helical shape of the flagellar filament, which is important for its proper function.
The N- and C-termini of flagellin form the inner core of the flagellin protein, and is responsible for flagellin's
ability to polymerize into a filament. The central portion of the protein makes up the outer surface of the flagellar
filament. While the termini of the protein is quite similar among all bacterial flagellins, the central portion is
wildly variable.

Tubulin in molecular biology can refer either to the tubulin protein superfamily of globular proteins, or one of the
member proteins of that superfamily. Tubulin is also used to specifically refer
to α-tubulin and β-tubulin, the proteins that make up microtubules in
eukaryotic cells. Microtubules are assembled from dimers of α- and β-tubulin.
To form microtubules, the dimers of α- and β-tubulin bind to GTP and
assemble onto the (+) ends of microtubules while in the GTP-bound state. The
β-tubulin subunit is exposed on the plus end of the microtubule while the α-
tubulin subunit is exposed on the minus end. After the dimer is incorporated
into the microtubule, the molecule of GTP bound to the β-tubulin subunit
eventually hydrolyzes into GDP through inter-dimer contacts along the
microtubule protofilament. Whether the β-tubulin member of the tubulin dimer
is bound to GTP or GDP influences the stability of the dimer in the
microtubule. Dimers bound to GTP tend to assemble into microtubules, while
dimers bound to GDP tend to fall apart; thus, this GTP cycle is essential for the dynamic instability of the
microtubule.

Dynein is a motor protein (also called molecular motor or motor molecule) in cells which converts the chemical
energy contained in ATP into the mechanical energy of movement. Dynein transports various cellular cargo by
"walking" along cytoskeletal microtubules towards the minus-end of the microtubule, which is usually oriented
towards the cell center. Thus, they are called "minus-end directed motors." This form of transport is known as
retrograde transport. In contrast, kinesins, which are motor proteins that move toward the microtubules' plus end,
are called plus-end directed motors.

A kinesin is a protein belonging to a class of motor proteins found in eukaryotic cells. Kinesins move along
microtubule (MT) filaments, and are powered by the hydrolysis of adenosine triphosphate (ATP) (thus kinesins are
ATPases). The active movement of kinesins supports several cellular functions including mitosis, meiosis and
transport of cellular cargo, such as in axonal transport. Most kinesins walk towards the positive end of a
microtubule, which, in most cells, entails transporting cargo from the centre of the cell towards the periphery.
This form of transport is known as anterograde transport. In contrast, dyneins are motor proteins that move
toward the microtubules' negative end.
Remember that negatively charged phosphates groups of nucleotides contribute to the overall negative charge of
DNA molecule. Thus, in order for histones to bind to DNA, they must be positively charged. Look back again at
the amino acid table in question 1 and notice that arginine, lysine, and histidine are all positively charged amino
acids. Thus you should choose D. Lys.

Pepsin hydrolyzes peptide bonds only. Thus, you can rule out answers A and E. If pepsin hydrolysed proteins to
free amino acids, then there would be no need for trypsin to be secreted from pancreas in small intestine. Thus you
can rule out statement B. Pepsin is most efficient in cleaving peptide bonds between hydrophobic and preferably
aromatic amino acids such as phenylalanine, tryptophan, and tyrosine. As various proteins have different
sequences of amino acids, you can predict that pepsin will cleave proteins into peptides of VARIED length. Thus I
would choose answer D.

The answer should be C.


Non-cyclic photophosphorylation produces ATP using the energy from excited electrons provided by photosystem
II. As electrons pass through the non-cyclic pathway, they do not return to the original photosystem. This does not
create a cycle, hence the name non-cyclic.
1. Photolysis provides H+ ions to replace those lost in the photosystems. The excited electrons provide energy for
a proton pump to actively transport additional H+ into the thylakoid. The high concentration of H+ diffuse past
ATP synthase as they pass out of the membrane to the lower H+ concentration. The energy created as H+ passes
the ATP synthase forms ATP. Diffusion of H+ ions from high to low concentration through ATP synthase to form
ATP is called chemiosmosis.
2. The electrons provided by non-cyclic photophosphorylation are boosted a 2nd time to a higher energy level by
photosystem I to then create 2 NADPH.

Statement B is correct. The pericycle is a cylinder of parenchyma or sclerenchyma cells that lies just inside the
endodermis and is the outer most part of the stele of plants. In eudicots, it also has the capacity to produce lateral
roots. Branch roots arise from this primary meristem tissue. In plants undergoing secondary growth, the pericycle
contributes to the vascular cambium often diverging into a cork cambium. Since it has the capacity to produce
lateral roots, the pericycle also, therefore, regulates the formation of lateral roots. Monocot roots rarely branch,
but can, and this branch will originate from the pericycle.
Cross section of monocot root:

Cross section of dicot root:


Statement C is also correct because in eustele the primary vascular tissue consists of vascular bundles, usually in
one or two rings around the pith. In addition to being found in stems, the eustele appears in the roots of monocot
flowering plants. Pith is the innermost region of the root representing the central axis. It is composed of few
loosely arranged parenchyma cells.

Statement D is also correct and you can confirm this


by looking at the picture above.
Statement A is incorrect because in eudicot roots, the
centre is formed by star-shaped xylem, not phloem.
Thus you should choose answer A.

Photosystems are functional and structural units of protein complexes involved in photosynthesis that together
carry out the primary photochemistry of photosynthesis: the absorption of light and the transfer of energy and
electrons. They are found in the thylakoid membranes of plants, algae and cyanobacteria (in plants and algae
these are located in the chloroplasts), or in the cytoplasmic membrane of photosynthetic bacteria. There are two
kinds of photosystems: II and I, respectively.
Statement B seems to be the only correct one.

At the heart of a photosystem lies the reaction center, which is an enzyme that uses light to reduce molecules. In a
photosystem, this reaction center is surrounded by light-harvesting complexes that enhance the absorption of light.
Two families of reaction centers in photosystems exist: type I reaction centers (such as photosystem I (P700) in
chloroplasts and in green-sulphur bacteria) and type II reaction centers (such as photosystem II (P680) in
chloroplasts and in non-sulphur purple bacteria).
Each photosystem can be identified by the wavelength of light to which it is most reactive (700 and 680
nanometers, respectively for PSI and PSII in chloroplasts), the amount and type of light-harvesting complexes
present and the type of terminal electron acceptor used.

Type I photosystems use ferredoxin-like iron-sulfur cluster proteins as terminal electron acceptors, while type II
photosystems ultimately shuttle electrons to a quinone terminal electron acceptor. One has to note that both
reaction center types are present in chloroplasts and cyanobacteria, working together to form a unique
photosynthetic chain able to extract electrons from water, creating oxygen as a byproduct.

For oxygenic photosynthesis, both photosystems I and II are required. Oxygenic photosynthesis can be performed
by plants and cyanobacteria; cyanobacteria are believed to be the progenitors of the photosystem-containing
chloroplasts of eukaryotes. Photosynthetic bacteria that cannot produce oxygen have a single photosystem called
BRC, bacterial reaction center.

The photosystem I was named "I" since it was discovered before photosystem II, but this does not represent the
order of the electron flow.
When photosystem II absorbs light, electrons in the reaction-center chlorophyll are excited to a higher energy
level and are trapped by the primary electron acceptors. To replenish the deficit of electrons, electrons are
extracted from water by a cluster of four Manganese ions in photosystem II and supplied to the chlorophyll via a
redox-active tyrosine.

Photoexcited electrons travel through the cytochrome b6f complex to photosystem I via an electron transport
chain set in the thylakoid membrane. This energy fall is harnessed, (the whole process termed chemiosmosis), to
transport hydrogen (H+) through the membrane, to the lumen, to provide a proton-motive force to generate ATP.
The protons are transported by the plastoquinone. If electrons only pass through once, the process is termed
noncyclic photophosphorylation.
When the electron reaches photosystem I, it fills the electron deficit of the reaction-center chlorophyll of
photosystem I. The deficit is due to photo-excitation of electrons that are again trapped in an electron acceptor
molecule, this time that of photosystem I.
ATP is generated when the ATP synthase transports the protons present in the lumen to the stroma, through the
membrane. The electrons may either continue to go through cyclic electron transport around PS I or pass, via
ferredoxin, to the enzyme NADP+ reductase. Electrons and hydrogen ions are added to NADP+ to form NADPH.
This reducing agent is transported to the Calvin cycle to react with glycerate 3-phosphate, along with ATP to form
glyceraldehyde 3-phosphate, the basic building-block from which plants can make a variety of substances.

As for statement C, Fluorescence Resonance Energy Transfer (FRET) is a special technique to gauge the distance
between two chromophores, called a donor-acceptor pair. The limitation of FRET is that this transfer process is
effective only when the separating distance of donor-acceptor pair is smaller than 10 nanometers. However, FRET
is a highly distance-dependent phenomenon and thus has become a popular tool to measure the dynamic activities
of biological molecules within nanoscale.
FRET is the acronym of the Förster (Flourescence) Resonance Energy Transfer. The Förster energy transfer is the
phenomenon that an excited donor transfers energy (not an electron) to an acceptor group through a non-
radiative process. This process is highly distance-dependent, thus allowing one to probe biological structures.
One common application is simply to measure the distance between two positions of interest on a big molecule,
generally a biological macromolecule, by attaching appropriate donor-acceptor groups to the big one. If the big
molecule only involves one donor and one acceptor group, the distance between the donor and the acceptor can
be easily measured if there is no conformational change within this process. Besides, if the molecule has a huge
conformational change, one may also measure the dynamical activities between two sites on this macromolecule
such as protein interactions. Today, this technique is widely applied in many fields such as single-molecule
experiments, molecular imaging, molecular motors, biosensors and DNA mechanical movements. FRET is also
called the "Spectroscopic Ruler" because of its intrinsic convenience.
The picture illustrates eudicot root (notice
star-shaped xylem and root hairs).
16. E. The pericycle is a cylinder of
parenchyma or sclerenchyma cells that lies
just inside the endodermis and is the outer
most part of the stele of plants. The pericycle
is located between the endodermis and phloem
in plant roots. In dicot stems, it is situated
around the ring of vascular bundles in the
stele. Its main function is to produce lateral
roots.
17. C. Sugar transport occurs via phloem.
18. D. Water transport occurs via xylem.
19. E. Notice the presence of root hairs which
indicate that this specimen is from the area of
maturation.

Root Zones
In most vascular plants, roots are
underground structures that anchor the plant and provide a means to absorb the nutrients and water needed for
growth of the plant body. New root tips grow continuously throughout the life of the plant and provide the surfaces
through which most of the nutrients and water move. Roots are used as storage organs for the food materials
produced by the shoots. The major functions of roots, thus, can be summarized simply as absorption, conduction,
storage, and anchorage.
As cells are added to the tip by repeated cell divisions, a young root elongates and leaves behind cells that
differentiate and become the primary roots of the plant. Four areas of the young root traditionally are recognized,
but except for the terminal area, are not distinctly separate. Their descriptive names are only partially correct in
describing the activities taking place in each area. These regions, starting at the tip and moving upwards towards
the stem, are the root cap, zone of active cell division, zone of cell elongation, and zone of maturation.
The first two are compacted in the first centimeter or less of the axis with the latter two no more than 4–5
centimeters from the tip. Only the root cap and the cell division regions actually move through the soil. After cells
start to elongate and mature, no further extension takes place, and the root is stationary for the rest of its life.

Root cap
The root cap is a cup-shaped, loosely cemented mass of parenchyma cells that covers the tip of the root. As cells
are lost among the soil particles, new ones are added from the meristem behind the cap. The cap is a unique
feature of roots; the tip of the stem has no such structure. From its shape, structure, and location, its primary
function seems clear: It protects the cells under it from abrasion and assists the root in penetrating the soil.
Phenomenal numbers of cap cells are produced to replace those worn off and lost as root tips push through the
soil.
The movement is assisted by a slimy substance, mucigel, which is produced by cells of the root cap and epidermis.
The mucigel:
• Lubricates the roots.
• Contains materials that are inhibitory to roots of other species.
• Influences ion uptake.
• Attracts beneficial soil microorganisms.
• Glues soil particles to the roots thereby improving the soil-plant contact and facilitating water movement
from the soil into the plant.
• Protects the root cells from drying out.

Root cap cells sense light in some as yet unexplained way and direct root growth away from light. The root cap
also senses gravity to which roots respond by growing downward, bringing them into contact with the soil, the
reservoir of nutrients and water used by plants. The root cap also responds to pressures exerted by the soil
particles.

Zone of cell division


An apical meristem lies under and behind the root cap and, like the stem apical meristem, it produces the cells that
give rise to the primary body of the plant. Unlike the stem meristem, it is not at the very tip of the root; it lies
behind the root cap. Between the area of active division and the cap is an area where cells divide more slowly, the
quiescent center. Most cell divisions occur along the edges of this center and give rise to columns of cells
arranged parallel to the root axis. The parenchyma cells of the meristem are small, cuboidal, with dense
protoplasts devoid of vacuoles and with relatively large nuclei.
The apical meristem of the root organizes to form the three primary meristems: protoderm, which gives rise to the
epidermis; procambium, which produces xylem and phloem; and the ground meristem, which produces the cortex.
Pith, present in most stems and produced from the ground meristem, is absent in most dicot (eudicot) roots, but is
found in many monocot roots.
Zone of cell elongation
The cells in this zone stretch and lengthen as small vacuoles within the cytoplasm coalesce and fill with water.
One or two large vacuoles occupy almost all of the cell volume in fully elongated cells. Cellular expansion in this
zone is responsible for pushing the root cap and apical tip forward through the soil.
Zone of maturation
The elongating cells complete their differentiation into the tissues of the primary body in this zone. It is easily
recognized because of the numerous root hairs that extend into the soil as outgrowths of single epidermal cells.
They greatly increase the absorptive surface of roots during the growth period when large amounts of water and
nutrients are needed. An individual root hair lives for only a day or two, but new ones form constantly nearer the
tip as old ones die in the upper part of the zone.

You are given a cross section of woody stem. Notice the difference between summer wood and spring wood.
Because stems are circular, there is no chance to escape if you go up or down in the diagram above (rule out
answers A and D). Because we do not know the height of a plant, going vertically up would probably be the
longest way (rule out choice E). Because darker summer wood (which is closer to the bark and consequently
would be the quickest way out) is on the right, this means that you should go C. Right.
The answer should be C. Plants have critical value for hours of darkness, but not for number of flashes.
Short-day (or long-night) plants experiencing a long night will not flower if exposed briefly to light sometime
during the night.
Far-red light cancels the effect of red light. Shortening of night
length by red light (R) can be negated by a flash of far-red light
(FR). When this occurs, the plant perceives no interruption in night
length. No matter how many times red light is flashed, as long as it
is followed by far-red light the effects of red light are canceled.
This works in both short-day and long-day plants.

Short-day plants flower when daylight is less than a critical length. They flower in the
late summer, fall, or early winter. Examples: chrysanthemums (“mums”), poinsettias,
some soybeans.
In essence it is the period of darkness that determines if a plant will flower. It's All About
Night Length, Not Day Length!
Key discovery: photoperiodism has nothing to do with day length—it is completely
dependent on a critical night length. Summary of research using the cocklebur plant:
The critical night length for the cocklebur is 8 hours: as long as the cocklebur plant has
at least 8 hours of continuous darkness, it will flower.
The answer should be D. Nitric oxide is the smallest of all molecules mentioned above and thus is capable of
passing easily through the membranes. This means that storage within vesicles is impossible.
Since it was first discovered to play a role in the dilation of blood vessels, many new roles for nitric oxide (NO)
have been discovered. Nitric oxide has been found to be produced by virtually every cell type in the body and
plays an important role in controlling the normal function of cells as well as in regulating larger scale
processes such as the nervous and immune systems.

The answer should be A. The heart’s action is mainly contraction. Calcium ions are essential for excitation
contraction coupling. This is the process linking electrical excitation to contraction. Calcium has an essential
role in this process; a raised intracellular calcium concentration is the trigger that activates contraction. An
understanding of calcium handling is essential to understanding the function of the heart. The intracellular
calcium ion concentration in the cardiac myocyte at rest is 0.0001 mM litre−1 and that in the extracellular
fluid is 1.2 mM litre−1. During the plateau phase of the action potential, calcium ions flow down this steep
concentration gradient and enter the myocyte. Most of this calcium enters through the L-type channels, located
primarily at sarcolemmal/sarcoplasmic reticulum junctions. The influx of calcium triggers the release of
further calcium from the sarcoplasmic reticulum via ryanodine receptors. This calcium-triggered calcium
release is in contrast to
skeletal muscle, where the
action potential triggers
calcium release directly.
Free intracellular calcium
interacts with the C subunit
of troponin. This leads to a
configuration change in the
troponin/tropomyosin
complex, allowing actin to
interact with myosin. Cross
bridge cycling occurs,
leading to a shortening of
the sarcomere and resultant
muscular contraction. As
intracellular calcium
concentrations decrease
during repolarization,
calcium dissociates from
troponin as intracellular
calcium concentration
decreases, resulting in relaxation. Diastolic relaxation is an active (ATP-dependent) process. Calcium
transport out of the cytosol occurs via a sarcoplasmic reticulum Ca2+-ATPase, through sarcolemmal
Na+/Ca2+ exchange, via a sarcolemmal Ca2+-ATPase, and finally by utilizing a mitochondrial Ca2+ uniport.
The strength of a contraction may be varied by increasing the amount of free intracellular calcium, by altering
the sensitivity of the myofilaments to calcium, or both. The latter occurs during stretching of the myofilaments
and is responsible for the Frank–Starling mechanism.

Potassium is essential for contraction. Potassium is specifically needed for voltage-gated potassium channels to
work in the outer membranes of cardiac muscle cells. These channels open in response to a change in voltage and
are responsible for terminating action potentials and contractions while initiating repolarization. There are two
main types of K+ channels but all have a basic common function: the creation of a transmembrane "leak" of
potassium ions out of the cell (efflux), which is responsible for hyperpolarization.

The correct answer is B - the T cells will express both MHC a & MHC b on their surface but will only be exposed
to MHC a on the thymic epithelial cells and thus will be restricted only by MHC a (statements C and D are thus
incorrect).
A haplotype is a set of DNA variations, or polymorphisms, that tend to be inherited together. A haplotype is a
group of genes within an organism that was inherited together from a single parent. The word "haplotype" is
derived from the word "haploid," which describes cells with only one set of chromosomes, and from the word
"genotype," which refers to the genetic makeup of an organism. A haplotype can describe a pair of genes inherited
together from one parent on one chromosome, or it can describe all of the genes on a chromosome that was
inherited together from a single parent. This group of genes was inherited together because of genetic linkage, or
the phenomenon by which genes that are close to each other on the same chromosome are often inherited
together.
MHC-restricted antigen recognition, or MHC restriction, refers to the fact that a given T cell will recognize a
peptide antigen only when it is bound to a host body's own MHC molecule. Normally, as T cells are stimulated
only in the presence of self-MHC molecules, antigen is recognized only as peptides bound to self-MHC molecules.
MHC restriction is particularly important when primary lymphocytes are developing and differentiating in the
thymus or bone marrow. It is at this stage that T cells die by apoptosis if they express high affinity for self-antigens
presented by an MHC molecule or express too low affinity for self MHC. This is ensured through two distinct
developmental stages: positive selection and negative selection. Positive selection ensures that any cells with a
high enough affinity for peptide bound MHC survive. Negative selection induces death in cells which bind MHC
too strongly.

Multiple sclerosis destroys myelin sheath and myelin is made by oligodendrocytes and Shwann cells. Thus, you
should choose answers C and E.

Location Name Description


The most abundant type of macroglial cell in the CNS, astrocytes (also
called astroglia) have numerous projections that anchor neurons to their
blood supply. They regulate the external chemical environment of
neurons by removing excess ions, the notable one being potassium, and
recycling neurotransmitters released during synaptic transmission. The
current theory suggests that astrocytes may be the predominant
"building-blocks" of the blood–brain barrier. Astrocytes may regulate
vasoconstriction and vasodilation by producing substances such as
arachidonic acid, whose metabolites are vasoactive.
Astrocytes signal each other using calcium. The gap junctions (also
known as electrical synapses) between astrocytes allow the messenger
molecule IP3 to diffuse from one astrocyte to another. IP3 activates
calcium channels on cellular organelles, releasing calcium into the
CNS Astrocytes
cytoplasm. This calcium may stimulate the production of more IP3.
The net effect is a calcium wave that propagates from cell to cell.
Extracellular release of ATP, and consequent activation of purinergic
receptors on other astrocytes, may also mediate calcium waves in some
cases.
In general, there are two types of astrocytes, protoplasmic and fibrous,
similar in function but distinct in morphology and distribution.
Protoplasmic astrocytes have short, thick, highly branched processes
and are typically found in gray matter. Fibrous astrocytes have long,
thin, less branched processes and are more commonly found in white
matter.
It has recently been shown that astrocyte activity is linked to blood
flow in the brain, and that this is what is actually being measured in
fMRI. They also have been involved in neuronal circuits playing an
inhibitory role after sensing changes in extracellular calcium.
Oligodendrocytes are cells that coat axons in the central nervous
system (CNS) with their cell membrane, forming a specialized
CNS Oligodendrocytes membrane differentiation called myelin, producing the so-called
myelin sheath. The myelin sheath provides insulation to the axon that
allows electrical signals to propagate more efficiently.[18]
Ependymal cells, also named ependymocytes, line the spinal cord and
the ventricular system of the brain. These cells are involved in the
CNS Ependymal cells creation and secretion of cerebrospinal fluid (CSF) and beat their cilia
to help circulate the CSF and make up the blood-CSF barrier. They are
also thought to act as neural stem cells.
Radial glia cells arise from neuroepithelial cells after the onset of
neurogenesis. Their differentiation abilities are more restricted than
those of neuroepithelial cells. In the developing nervous system, radial
glia function both as neuronal progenitors and as a scaffold upon which
CNS Radial glia newborn neurons migrate. In the mature brain, the cerebellum and
retina retain characteristic radial glial cells. In the cerebellum, these are
Bergmann glia, which regulate synaptic plasticity. In the retina, the
radial Müller cell is the principal glial cell, and participates in a
bidirectional communication with neurons.
Similar in function to oligodendrocytes, Schwann cells provide
myelination to axons in the peripheral nervous system (PNS). They
PNS Schwann cells
also have phagocytotic activity and clear cellular debris that allows for
regrowth of PNS neurons.
Satellite glial cells are small cells that surround neurons in sensory,
sympathetic, and parasympathetic ganglia. These cells help regulate the
external chemical environment. Like astrocytes, they are
PNS Satellite cells interconnected by gap junctions and respond to ATP by elevating
intracellular concentration of calcium ions. They are highly sensitive to
injury and inflammation, and appear to contribute to pathological
states, such as chronic pain
Are found in the intrinsic ganglia of the digestive system. They are
PNS Enteric glial cells thought to have many roles in the enteric system, some related to
homeostasis and muscular digestive processes.
I would choose D. 55% is too small number whereas 95% is too high number. Muscle contractions produce heat
and as much as 70% of body heat is produced by energy produced in muscle tissue. Blood is an essential element
in temperature control during exercise, taking heat from the body core and working muscles and redirecting it to
the skin when the body is overheating. When the internal heat of the body reaches too low a level thermoreceptors
in the skin relay a message to the hypothalamus in the brain. In response to this signal, the skeletal muscles
contract and relax in an involuntary manner (shivering) increasing muscle activity to generate heat. In turn,
muscles are also responsive to exterior heat - cold air increases muscle tone, and hot conditions have a relaxing
effect on muscles.

Depending on whether they are initiated through locomotion and intentional movement of the muscles,
thermogenic methods can be classified as one of the following:
• Exercise-associated thermogenesis (EAT)
• Non-exercise activity thermogenesis (NEAT)
• Diet-induced thermogenesis (DIT)

Shivering
One method to raise temperature is through shivering. It produces heat because the conversion of the chemical
energy of ATP into kinetic energy causing some of the energy to show up as heat. It is not 100% efficient, meaning
while some of the energy becomes heat, a portion is transferred to the kinetic energy that produces its
characteristic muscular twitches. No productive movement is produced in shivering because antagonistic muscle
pairs are simultaneously activated. Shivering is the process by which the body temperature of hibernating
mammals (such as some bats and ground squirrels) is raised as these animals emerge from hibernation.
Non-shivering
Non-shivering thermogenesis occurs in brown adipose tissue (brown fat) that is present in all eutherians (swine
being the only exception currently known). Brown adipose tissue has a unique protein (uncoupling protein-1) that
allows the uncoupling of protons moving down their mitochondrial gradient from the synthesis of ATP, thus
allowing the energy to be dissipated as heat.
In this process, substances such as free fatty acids (derived from triacylglycerols) remove purine (ADP, GDP and
others) inhibition of thermogenin (uncoupling protein-1), which causes an influx of H+ into the matrix of the
mitochondrion and bypasses the ATP synthase channel. This uncouples oxidative phosphorylation, and the energy
from the proton motive force is dissipated as heat rather than producing ATP from ADP, which would store
chemical energy for the body's use. Thermogenesis can also be produced by leakage of the sodium-potassium
pump and the Ca2+ pump. Thermogenesis is contributed to by futile cycles, such as the simultaneous occurrence
of lipogenesis and lipolysis or glycolysis and gluconeogenesis.
The low demands of thermogenesis mean that free fatty acids draw, for the most part, on lipolysis as the method of
energy production.
The answer should be C. III -> VI -> II -> I-> V->IV.

D is the most probable and reasonable. Remember that it is believed a human must drink 2 L of water every day
and in order for homeostasis to be in balance, the same amount must be removed by kidneys.
As the filtrate travels through the nephron, most of its components are reabsorbed to varying degrees, depending
on the need to adjust blood chemistry and the kidney's ability to reabsorb nutrients and other molecules. The cells
of the nephron may secrete ions and drugs as well.
Since urine output is only 1 - 2 L/day, it is apparent that most of the water filtered, as well as required nutrients
and electrolyes are returned to circulation. The amount of urine produced varies with dietary intake, exercise and
pregnancy.
Answer should be E. Remember that birds excrete uric acid in order to save water.

Statement E is correct because portal vein can bleed profusely, thus wounds are life-threatening.
Statement C is correct because pancreas is mainly found on the left side as well as the centre of the body.
Statement B is correct because colon has four parts: ascending, descending, transverse, and sigmoid parts, which
surrounds small intestines like a square, therefore, colon could have been affected as well.
Statement A is correct because pancreas is located in the upper part of the abdomen, primarily in the epigastrium
and extending into the left hypogastrium. It lies retroperitoneally, traversing the transpyloric line at the level of
the L1 and L2 vertebrae. The transverse colon is the longest and most movable part of the colon. It crosses the
abdomen from the ascending colon at the hepatic or right colic flexure with a downward convexity to the
descending colon where it curves sharply on itself beneath the lower end of the spleen forming the splenic or left
colic flexure. In its course, it describes an arch, the concavity of which is directed backward and a little upward. It
is almost completely invested by peritoneum, and is connected to the inferior border of the pancreas by a large
and wide duplicature of that membrane, the transverse mesocolon.
It is in relation, by its upper surface, with the liver and gall-bladder, the greater curvature of the stomach, and the
lower end of the spleen; by its under surface, with the small intestine; by its anterior surface, with the anterior
layers of the greater omentum and the abdominal wall; its posterior surface is in relation from right to left with
the descending portion of the duodenum, the head of the pancreas, and some of the convolutions of the jejunum
and ileum.
Statement D is incorrect because portal vein collects blood from the stomach via gastric veins, not supplies it:

Neuromuscular junction is a chemical synapse formed by the contact between the presynaptic terminal of a
motor neuron and the postsynaptic membrane of a muscle fiber. It is at the neuromuscular junction that a
motor neuron is able to transmit a signal to the muscle fiber, causing it to contract.

Synaptic transmission at the neuromuscular junction begins when an action potential reaches the presynaptic
terminal of a motor neuron, which activates voltage-dependent calcium channels to allow calcium ions to enter
the neuron. Calcium ions bind to sensor proteins (synaptotagmin) on synaptic vesicles, triggering vesicle fusion
with the cell membrane and subsequent neurotransmitter release from the motor neuron into the synaptic cleft.
In vertebrates, motor neurons release acetylcholine (ACh), a small molecule neurotransmitter, which diffuses
across the synaptic cleft and binds to nicotinic acetylcholine receptors (nAChRs) on the cell membrane of the
muscle fiber, also known as the sarcolemma. nAChRs are ionotropic receptors, meaning they serve as ligand-
gated ion channels. The binding of ACh to the receptor can depolarize the muscle fiber, causing a cascade that
eventually results Acetylcholine is a neurotransmitter synthesized from dietary choline and acetyl-CoA (ACoA),
and is involved in the stimulation of muscle tissue in vertebrates as well as in some invertebrate animals. in
muscle contraction.

If curare competes with acetylcholine, this means that whenever it binds to ACh receptors, Ach is unable to
activate signal transduction pathway and exert its effect, i.e. muscle contraction.
Flaccid paralysis occurs when the muscle cannot contract at all. The muscle stays weak and floppy.
Spastic paralysis occurs when the muscle stays in contraction. You still cannot move the muscle properly, but in
this case, the muscle is too rigid.
Insecticides allow excess Ach to accumulate and cause spastic paralysis. Ach accumulates and acts as a
constant stimulus. The muscles go into spastic paralysis and the insect suffocates.
Curare blocks the Ach receptors, causing flaccid paralysis. Thus the answer should be D.

The answer should be C. III->V->I->II->IV->VI.


The correct answer is A. Environmental hypoxia ultimately affects the oxygen delivery to tissues. This is a
cumulative outcome of physiological transport system to hold oxygen, its flow rate and the partial pressure
gradient between blood and the tissues that drives oxygen diffusion. For instance, the driving force for gas-
exchange at the alveolar-capillary junction is driven by partial pressure gradient of O2 and CO2 in the
alveolar air and in the blood. At high altitude, due to the fall in ambient O2 partial pressure, the alveolar
partial pressure of O2 falls; this further reduces its gradient against the partial pressure of O2 in deoxygenated
blood leading to reduced diffusion of O2 into the arterial blood. Similarly, as the pressure differential between
venous CO2 and alveolar CO2 decreases, the driving force for the removal of CO2 diminishes and thus a
higher amount is retained in the arterial blood at high altitude. This lower oxygen partial pressure (hypoxia)
and raised CO2 partial pressure (hypercapnia) in the arterial blood trigger a series of compensatory
adjustments by the various organ systems.

Statement B is incorrect because due to the fall in ambient O2 partial pressure, the alveolar partial pressure of
O2 falls and this means that respiratory system must increase in its activity in order to maintain adequate
levels of oxygen in the blood. The immediate effect of exposure to high altitude hypoxia is faster respiration
rate, i.e., pulmonary hyperventilation, which is merely a first visible symptom of deep-seated respiratory
imbalances, acid–base imbalance and other complex metabolic disturbances. Hyperventilation at high altitude
is controlled by the stimulus of peripheral chemoreceptor organs along the aorta and the carotid sinus. The
glomus cells are sensitive to changes in arterial O2 partial pressure and the raised CO2 partial pressure, and
signal to the respiratory centres in the central nervous system to increase the rate and depth of ventilation or
Hypoxic Ventilator Response (HVR). In response to peripheral chemoreceptors, the respiratory centre is
triggered in cerebral pons and medulla and the signals are relayed to the diaphragm, intercostal muscles and
stretch receptors of the lungs to facilitate HVR. This ventilator response sets in within a few hours of hypoxic
exposure reinstating the driving force of diffusion by raising alveolar ventilation by 25–30% leading to an
increased partial pressure of oxygen in the alveolar spaces.

Statement D is incorrect because due to compensatory mechanisms (increased RBC number, Hb, 2,3-DPG),
adaptation after a week will lead to maintenance of adequate blood pO2. However, compensatory mechanisms
will not restore normal blood pO2.

Statement E is incorrect because hematocrit is the volume percentage (%) of red blood cells in blood. It is
normally 45% for men and 40% for women. If the body adapts to reduced oxygen availability by synthesizing
more RBCs, this means that haematocrit will increase during adaptation period.

Because more RBCs are synthesized, haematocrit is increased and this means that blood viscosity is also
elevated. This in turn means that blood pressure increases, not decreases, therefore you can rule out statement
C.

Pulmonary embolism is a blockage in one of the pulmonary arteries in your lungs. In most cases, pulmonary
embolism is caused by blood clots that travel to the lungs from the legs or, rarely, other parts of the body (deep
vein thrombosis). If we have a blockage in pulmonary arteries in the left lung, then the blood will accumulate
upstream of the blockage, that is, in the right ventricle.

Pulmonary embolism can also lead to pulmonary hypertension, a condition in which the blood pressure in your
lungs and in the right side of the heart is too high. When you have obstructions in the arteries inside your lungs,
your heart must work harder to push blood through those vessels. This increases the blood pressure within
these vessels and the right side of the heart, which can weaken your heart.

Pulmonary embolism reduces the cross-sectional area of the pulmonary vascular bed, resulting in an increment
in pulmonary vascular resistance, which, in turn, increases the right ventricular afterload. If the afterload is
increased severely, right ventricular failure may ensue.

Statement D is correct because hypertrophy is an adaptive response to a long-term increase in pressure.


Individual muscle cells grow larger (in thickness) and change to drive the increased contractile force required
to move the blood against greater resistance. Dilatation is a stretching (in length) of the ventricle in response
to acute increased pressure, such as when caused by a pulmonary embolism.

Statement A is incorrect because absorption of nutrients occurs in jejunum, a part of small intestine. The large
intestine is concerned with absorption of water, minerals, vitamins, and formation of faeces.

Statement B is incorrect because parmesan crème brulee is made of cheese and eggs that contain very small
amounts of carbohydrates. You should know that in mouth alpha amylase starts to digest starch which is not
present in the appetizer mentioned above.

Statement C is correct because stomach contains hydrochloric acid as well as protease pepsin involved in
protein degradation.

Statement D is incorrect because even though stomach contains gastric lipase, the main site of lipid digestion is
duodenum where bile acids emulsify fats and pancreatic lipase that can hydrolyze fats.

Statement E is correct because by producing faeces, large intestine reabsorbs water to make them solid.

Thus correct answers are C and E.


Statement A is correct because without circulatory
system, gas exchange would be ineffective and gases
would not be exchange in distant parts of the body.

Statement B is incorrect because only birds have air


sacs.

Depending upon the species, the bird has seven or


nine air sacs. The air sacs include:
Two posterior thoracic
Two abdominal
Two anterior thoracic
Two cervical (these are not present in some species)
One interclavicular

Statement C is incorrect because in mammalian lungs,


the exchange of oxygen and carbon dioxide occurs in microscopic sacs in the lungs, called 'alveoli.' In the
avian lung, the gas exchange occurs in the walls of microscopic tubules, called parabronchi and air capillaries.
The mammalian respiratory system is partitioned homogeneously, so the functions of ventilation and gas
exchange are shared by alveoli and much of the lung volume.
The avian respiratory system is partitioned heterogeneously, so the functions of ventilation and gas exchange
are separate in the air sacs and the parabronchial lung, respectively.

Statement D is correct. Respiration in mammals is exactly the way we know it to be from humans, with lungs
and a diaphragm, which allows for negative pressure breathing. Mammals also utilize this breathing because it
is more efficient than positive pressure breathing, as air has to be forced into the body rather than creating a
pressure gradient where it naturally flows into and out of the lungs when required. Mammals require an
efficient supply of oxygen as they are considered the most advanced animals, and therefore require a lot of
processes to occur in their body. As the diaphragm contracts the volume in the lungs increases, resulting in a
lower pressure to be present in the lungs than outside, which results in the air to flow in. When the diaphragm
relaxes the volume decreases and therefore the pressure increases leading to the air to flow out. Birds do not
have a diaphragm; instead, air is moved in and out of the respiratory system through pressure changes in the
air sacs. Muscles in the chest cause the sternum to be pushed outward. This creates a negative pressure in the
air sacs, causing air to enter the respiratory system. Expiration is not passive, but requires certain muscles to
contract to increase the pressure on the air sacs and push the air out. Because the sternum must move during
respiration, it is essential that it is allowed to move freely when a bird is being restrained. Holding a bird "too
tight" can easily cause the bird to suffocate.

Birds use negative pressure breathing like reptiles and mammals, but they do not utilise a diaphragm to do this
as it will deflate their air sacs, which they require to keep filled. They use avian respiration which has a
unidirectional flow of air, meaning that the air flowing through will be largely fresh and not mixed with “old
air”, while in mammals there is a bidirectional flow. This makes sure that the oxygen supply in the body of a
bird is always high, which allows them to breath much more efficiently at much higher elevations than
mammals.

Statement E is incorrect. Birds, not mammals have more efficient respiratory system. Birds have a one-way
flow of air through their lungs. They don't have tiny air sacs called alveoli. Instead, they have tiny tubes called
air capillaries that the air flows through continuously and the benefit of doing that is that you always have
fresh air flowing through the lung, maintaining a very high concentration of oxygen up against the bloodstream
and therefore, you maximise the gradient for diffusion, pushing oxygen into the blood. If you were to dissect a
bird, what you would see is they have these combinations of lung tissue for want of a better word and air sacs.
Now the air sacs are in various parts of the body. They are called anterior and posterior air sacs as two
groups. When the bird breathes in, it moves various bones and muscles in order to increase volume of these air
sacs anteriorly and posteriorly, so they draw air into them. But, first of all, the air flows into the trachea and it
goes into the posterior air sac. The anterior air sacs which also increase in volume, they actually get filled up
by the air that's already in the bird’s lung. So in other words, they pull air through the lung tissue into their
anterior air sac. Then when the bird breathes out, exhales, it squeezes on these air sacs. This time, the
posterior air sac empties into the lung tissue and the anterior air sac empties into the bird’s trachea and then
out through its nostrils and its beak. So in this way, you've always got air going in one direction through the
lung. It’s always fresh air and therefore, you've always got a very high oxygen gradient taking air into the
tissue.

So you should pick B, C, and E.

The answer should be A. In a polygamous mating system, individuals of one or the other sex have more than
one mate during the breeding season. When males in the population mate with more than one female, it is
called polygyny (poly means "many," and gyne means "female"). Males compete for females, and this leads to
strong selection for traits that either attract females (for example, elaborate songs or calls, bright coloration,
and courtship displays) or allow males to compete effectively with other males (for example, aggressiveness,
large size, and fighting aids such as antlers). Polygyny is common in species where males are less likely to
provide parental care (and thus may increase their reproductive success by inseminating more females) or
where males are able to monopolize more than one female (if females or the resources they require are
spatially clumped). Mammalian mating systems are predominantly polygynous, in part because young develop
within and are then nursed by the female.
C. Habituation is an extremely simple form of learning, in which an animal, after a period of exposure to a
stimulus, stops responding. In our case prairie dogs stop responding (i.e. give alarm calls) after regular exposure
to stimulus (i.e. humans), which do not prey on prairie dogs and do not impose harm on them.

B. Cultural transmission, also known as cultural learning, is the process and method of passing on socially
learned information. Within a species, cultural transmission is greatly influenced by how adults socialize with
each other and with their young. Culture can be transmitted among animals through various methods, the most
common of which include imitation, teaching, and language. Imitation is one of the most prevalent modes of
cultural transmission in non-human animals, while teaching and language are much less widespread. In a study
on food acquisition techniques in meerkats (Suricata suricatta), researchers found evidence that meerkats learned
foraging tricks through imitation of conspecifics. The experimental setup consisted of an apparatus containing
food with two possible methods that could be used to obtain the food. Naïve meerkats learned and used the method
exhibited by the “demonstrator” meerkat trained in one of the two techniques. Although in this case, imitation is
not the clear mechanism of learning given that the naïve meerkat could simply have been drawn to certain features
of the apparatus from observing the “demonstrator” meerkat and from there discovered the technique on their
own.
In our case we have dolphins that teach their youngs, thus passing on experience which may help increase the
chances of survival.

Only individuals who have Aa or AA genotype are affected. Non-affected individuals should have aa genotype. If
we have two non-affected parent, the cross is aa x aa.
Statement A is the most probable because if replicated DNA has a mutation, it MAY be passed on to other cells,
resulting in the onset of the disease.
Statement B seems unlikely because transcription is a process during which mRNAs are produced and they are
often short-lived and not passed on to other cells. Thus if something goes wrong during transcription and the cell
then divides, it is unable to pass on a mutation to daughter cells.

Statement C is unlikely because even though translation has no proofreading mechanism, it cannot be passed on to
other cells and thus is isolated to one cell. This would not cause
a disease. Moreover, mRNAs and proteins are usually short
acting or their regulation is tightly controlled, therefore,
incorrect protein will be degraded quickly.
Statement E is incorrect because incomplete dominance results
in an intermediate phenotype between homozygous dominant
and homozygous recessive:

Since every mutant allele for an autosomal dominant disease is


expressed, and by definition a disease is a deleterious
phenotype, how do autosomal dominant diseases stay in the
population? Shouldn't they be eliminated by natural selection
against deleterious phenotypes? There appears to be four phenomena that maintain these deleterious alleles in the
population:
• Variable Expressivity
• Late Onset
• High Recurrent Mutation Rate
• Incomplete Penetrance
Variable expressivity. One example of variable expressivity is Marfan syndrome. Marfan syndrome is an
autosomal dominant disease caused by a mutation in collagen formation. It affects about 1/60,000 live births.
Symptoms of Marfan syndrome include skeletal, optical, and cardiovascular abnormalities. Skeletal abnormalities
include arachnodactyly (long fingers and toes), extreme lengthening of the long bones, scoliosis, rib and sternum
abnormalities, among others. Optical abnormalities almost always include ectopia lentis, a dislocation of the lens
into the anterior chamber of the eye. Cardiovascular abnormalities may be numerous and include possible
dissecting aneurysms, which are largely responsible for the shorter life span of Marfan syndrome patients as a
group. Each patient may express all of the symptoms, or only a few. That is variable expressivity. Each patient
with the mutant allele for Marfan syndrome expresses at least one of the symptoms, but the physician may have to
look closely. Almost all are taller than average, but a lot of non-Marfan individuals are tall. Almost all have long
fingers, but so do a lot of non-Marfan persons. Some may be very mildly affected and lead normal lives while
others, more severely affected, have a shorter life expectancy. The disease is maintained in the population through
recurrent mutations and the matings of less severely affected individuals with normal individuals. The extent of
severity of affected does not affect the severity of expression in the next generation, that is, the offspring of mildly
affected individuals range from mildly affected to severely affected, with equal probability.
Late onset. Some autosomal dominant diseases do not express themselves until later in life, well beyond the
reproductive years. The individuals who will develop the disease have passed the mutant allele along to their
offspring before they themselves know they are affected. In some cases even grandchildren are born before the
affected grandparent shows the first signs of the disease. Huntington disease, sometimes called Huntington's
Chorea because of the choreic movements expressed as the disease progresses, is a good example of a late onset
disease. Age of onset varies from the teens to the late sixties, with a mean age of onset between ages 35 and 45.
Nearly 100% of the individuals born with the defective allele will develop the disease by the time they are 70. The
disease is progressive with death usually occurring between four and twenty-five years after the first symptoms
develop. Emotional changes often are the first symptoms.
At the gene level, it is caused by the expansion of an unstable trinucleotide repeat sequence, CAG, in the coding
region of the gene. What is inherited at birth in Huntington disease is a gene with several repeats and the
instability that allows somatic recombination and extension. Somatic mutations introduced by the expansion of
trinucleotide repeat sequences do not have to occur in coding regions to produce a mutant allele. Other diseases,
such as myotonic dystrophy, an autosomal dominant disease where expression is delayed, result from molecular
defects at the gene level that are caused by the expansion of unstable trinucleotide sequences. (In the case of
myotonic dystrophy the sequence is CTG.) However, this unstable sequence lies in a non-translated region of the
gene. In both diseases the size of the inherited expansion correlates to the age of onset or the severity of disease,
but is not absolutely predictable on an individual basis. One cannot sequence the gene and precisely predict the
age of onset of Huntington disease.
High recurrent mutation rate. Achondroplasia is one of the major causes of dwarfism. Motor skills may not
develop as quickly as their normal siblings, but intelligence is not reduced. It occurs in about 1/10,000 live births.
Like many autosomal dominant diseases, individuals homozygous for the mutant allele do not survive to term.
Almost 85% of the cases are the result of new mutations, where both parents have a normal phenotype. The
mutation rate for achondroplasia may be as much as 10 times the "normal" mutation rate in humans. This high
recurrent mutation is largely responsible for keeping the mutant gene in the population at its present rate. Several
other autosomal dominant genetic diseases have high recurrent mutation rates but achondroplasia is probably the
best known.
Incomplete penetrance. Incomplete penetrance should never be confused with variable expressivity. In diseases
with variable expressivity the patient always expresses some of the symptoms of the disease and varies from very
mildly affected to very severely affected. In autosomal dominant diseases with incomplete penetrance, the person
either expresses the disease phenotype or he/she doesn't. Incomplete penetrance and variable expressivity are
phenomena associated only with dominant inheritance, never with recessive inheritance. The following pedigree
illustrates incomplete penetrance in a known autosomal dominant disease.
In the pedigree on the right, there is ample evidence for autosomal dominant inheritance:
• The disease is passed from the father (II-3)
to the son (III-5), this never happens with
X-linked traits.
• The disease occurs in three consecutive
generations, this never happens with
recessive traits.
• Males and females are affected, with
roughly the same probability.
• However, II-1 does not express the disease.
He must have inherited the mutant allele
because he passed it on to two children,
III-1 and III-3. II-1 is a classical example
of incomplete penetrance, he has the allele
for the disease but he does not express it.
Statement D may be possible in our case, however,
as we are told that neither parents, nor anyone
from their families has the disease, incomplete
penetrance is unlikely.
D. Achondroplasia is inherited in an autosomal dominant pattern, which means one copy of the altered gene in
each cell is sufficient to cause the disorder. About 80 percent of people with achondroplasia have average-size
parents; these cases result from new mutations in the FGFR3 gene. In the remaining cases, people with
achondroplasia have inherited an altered FGFR3 gene from one or two affected parents. Individuals who inherit
two altered copies of this gene typically have a severe form of achondroplasia that causes extreme shortening of
the bones and an underdeveloped rib cage. These individuals are usually stillborn or die shortly after birth from
respiratory failure.
The answer should be B.
Statement A is incorrect as this would indicate codominance.
Statement C is incorrect as this
would indicate complete
dominance.
Statement D is incorrect because
both parents are homozygous
and have identical alleles at
flower color locus, thus, crossing
over would make no difference
in gametes produced.
Incomplete dominance is a form
of intermediate inheritance in
which one allele for a specific
trait is not completely expressed
over its paired allele. This
results in a third phenotype in
which the expressed physical
trait is a combination of the
dominant and recessive
phenotypes.
Incomplete genetic dominance is similar to, but different from co-dominance. In co-dominance, an additional
phenotype is produced, however, both alleles are expressed completely.

The answer should be D. Homologous Structures are two structures that are similar in construction and often
share an evolutionary origin point. Occasionally they do not even have a similar appearance , shape or form. An
example of this would be the human arms, dolphin flippers, horse legs and bat wings.
Analogous structures are two structures that serve a common purpose but evolved separately in different species.
An example of this would be the flight capabilities of both birds and insects.

Convergent evolution describes the independent evolution of similar features in species of different lineages. The
two species came to the same function, flying, but did so separately from each other. They have "converged" on
this useful trait. Both sharks and dolphins have similar body forms, yet are only distantly related: sharks are fish
and dolphins are mammals. Such similarities are a result of both populations being exposed to the same selective
pressures. Within both groups, changes that aid swimming have been favored. Thus, over time, they developed
similar appearances (morphology), even though they are not closely related.
One of the most well-known examples of convergent evolution is the camera eye of cephalopods (e.g., octopus),
vertebrates (e.g., mammals), and cnidaria (e.g., box jellies). Their last common ancestor had at most a very simple
photoreceptive spot, but a range of processes led to the progressive refinement of this structure to the advanced
camera eye. There is, however, one subtle difference: the cephalopod eye is "wired" in the opposite direction, with
blood and nerve vessels entering from the back of the retina, rather than the front as in vertebrates.
Traits arising through convergent evolution are analogous structures, in contrast to homologous structures, which
have a common origin, but not necessarily similar
function.

Coevolution is the evolution of organisms of two or


more species in which each adapts to changes in the
other. Coevolution between herbivores and plants is
commonly seen in nature; for example, plants have
developed unique ways to fight off herbivores while, in
turn, herbivores have developed specialized features to
get around these defenses.

The answer should be D. The Poaceae are a large and nearly ubiquitous family of monocotyledonous flowering
plants. Grasslands are estimated to constitute 20% of the vegetation cover of the Earth. Poaceae live in many
other habitats, including wetlands, forests, and tundra.
Flowers of Poaceae are characteristically arranged in spikelets,
each spikelet having one or more florets. The spikelets are
further grouped into panicles or spikes. A spikelet consists of
two (or sometimes fewer) bracts at the base, called glumes,
followed by one or more florets. A floret consists of the flower
surrounded by two bracts, one external—the lemma—and one
internal—the palea. The flowers are usually hermaphroditic—
maize being an important exception—and anemophilous or
wind-pollinated. The perianth is reduced to two scales, called
lodicules, that expand and contract to spread the lemma and
palea; these are generally interpreted to be modified sepals.
The cross is M_bb x mmB_.
36 are B_.
17 are M_ B_.
19 are mm B_.
The ration is almost 1:1.

Choice A and D are incorrect because first parent would have mule-foot and white coat, not black as was stated
in the question.
If the parents are homozygous for both alleles, i.e. MMbb x mmBB, all offsprings are MmBb. Thus you can rule
our D immediately.
If both parents are heterozygotes for either foot or coat, that is Mmbb x mmBb so that F1 genotypes are as
follows: MmBb, mmBb, mmbb, Mmbb. As we got only two phenotypic classes, you can rule out choice C.
Now you are left with choice B, which is the correct one. If one parent is homozygous for both characteristics and
one is heterozygous, i.e. MMbb x mmBb or Mmbb x mmBB, then the offsprings will be MmBb and Mmbb or MmBb
and mmBb, respectively.

In the problem we have the following cross:


P: XAY x XaXa
F1: XAXa, XaY.
XA Y

Xa XAXa Xa Y
Xa XAXa Xa Y
From the Punett square you can see that 50% of children will have the dominant allele. However, the allele is not
100% penetrant and only in 70% of cases we will observe the disease, thus the percentage can be calculated
p=0.5*0.7=0.35 = 35%
The answer is B.
Let’s begin with E-H:

Then let’s add F and we have two possibilities to do so:


Now we can add G and we can do this in 4 different ways, one of which we can rule out:

We are told that E-F is 11cM and we need to find the distance between G-H, thus by looking at the remaining
three choices, notice that option 2) is incorrect because E-F is 5cM as is in option 4). Therefore, the only correct
one is option 1) and distance between G and H is 9+3=12 cM.
So the answers are A and C.
53. A.

54. B. The dominant photosynthetic organisms are phytoplankton, including photosynthetic bacteria, that drift
with the oceanic currents.
55. B. Unicellular algae, not bacteria live within the tissues of the corals, forming a mutualistic relationship that
provides the corals with organic molecules.
56. A.

57. A.
58. B. Many key traits of land plants also appear in some protists, primarily algae. For example, plants are
multicellular, eukaryotic, photosynthetic autotrophs, as are brown, red, and certain green algae. Plants have cell
walls made of cellulose, and so do green algae, dinoflagellates, and brown algae. And chloroplasts with
chlorophylls a and b are present in green algae, euglenids, and a few dinoflagellates, as well as in plants.
However, the charophytes are the only algae that share the following four distinctive traits with land plants,
strongly suggesting that they are the closest relatives of plants:
• Rings of cellulose-synthesizing proteins. The cells of both land plants and charophytes have distinctive circular
rings of proteins in the plasma membrane. These protein rings synthesize the cellulose microfibrils of the cell wall.
In contrast, noncharophyte algae have linear sets of proteins that synthesize cellulose.
• Peroxisome enzymes. The peroxisomes of both land plants and charophytes contain enzymes that help minimize
the loss of organic products resulting from photorespiration.
• Structure of flagellated sperm. In species of land plants that have flagellated sperm, the structure of the sperm
closely resembles that of charophyte sperm.
• Formation of a phragmoplast. Particular details of cell division occur only in land plants and certain
charophytes, including the genera Chara and Coleochaete. For example, in land plants and certain charophytes, a
group of microtubules known as the phragmoplast forms between the daughter nuclei of a dividing cell. A cell
plate then develops in the middle of the phragmoplast, across the midline of the dividing cell. The cell plate, in
turn, gives rise to a new cross wall that separates the daughter cells.

59. C. Many of the adaptations that appear to have emerged after land plants diverged from their algal relatives
facilitated survival and reproduction on dry land. Charophyte algae lack the four key traits of land plants
described in this figure: alternation of generations (with an associated trait of multicellular, dependent embryos),
walled spores produced in sporangia, multicellular gametangia, and apical meristems. This suggests that these
four traits were absent in the ancestor common to land plants and charophytes but instead evolved as derived
traits of land plants. Note that some of these traits are not unique to plants, having evolved independently in other
lineages. And not every land plant exhibits all four of these traits; certain lineages of plants have lost some traits
over time.

60. D. The presence of vessels in xylem has been considered to be one of the key innovations that led to the success
of the flowering plants. It was once thought that vessel elements were an evolutionary innovation of flowering
plants, but their absence from some basal angiosperms and their presence in some members of the Gnetales
suggest that this hypothesis must be re-examined; vessel elements in Gnetales may not be homologous with those
of angiosperms, or vessel elements that originated in a precursor to the angiosperms may have been subsequently
lost in some basal lineages (e.g., Amborellaceae, Tetracentraceae, Trochodendraceae, and Winteraceae).
Cronquist considered the vessels of Gnetum to be convergent with those of angiosperms.
Vessel-like cells have also been found in the xylem of Equisetum (horsetails), Selaginella (spike-mosses),
Pteridium aquilinum (bracken fern), Marsilea and Regnellidium (aquatic ferns), and the enigmatic fossil group
Gigantopteridales. In these cases, it is generally agreed that the vessels evolved independently. It is possible that
vessels may have appeared more than once among the angiosperms as well.

The answer should be A and B. The Hardy-Weinberg equilibrium is a principle stating that the genetic variation in
a population will remain constant from one generation to the next in the absence of disturbing factors. When
mating is random in a large population with no disruptive circumstances, the law predicts that both genotype and
allele frequencies will remain constant because they are in equilibrium.

The Hardy-Weinberg equilibrium can be disturbed by a number of forces, including mutations, natural selection,
nonrandom mating, genetic drift, and gene flow. For instance, mutations disrupt the equilibrium of allele
frequencies by introducing new alleles into a population. Similarly, natural selection and nonrandom mating
disrupt the Hardy-Weinberg equilibrium because they result in changes in gene frequencies. This occurs because
certain alleles help or harm the reproductive success of the organisms that carry them. Another factor that can
upset this equilibrium is genetic drift, which occurs when allele frequencies grow higher or lower by chance and
typically takes place in small populations. Gene flow, which occurs when breeding between two populations
transfers new alleles into a population, can also alter the Hardy-Weinberg equilibrium.

Because all of these disruptive forces commonly occur in nature, the Hardy-Weinberg equilibrium rarely applies
in reality. Therefore, the Hardy-Weinberg equilibrium describes an idealized state, and genetic variations in
nature can be measured as changes from this equilibrium state.
Hardy, Weinberg, and the population geneticists who followed them came to understand that evolution will not
occur in a population if seven conditions are met:

1. mutation is not occurring


2. natural selection is not occurring
3. the population is infinitely large
4. all members of the population breed
5. all mating is totally random
6. everyone produces the same number of offspring
7. there is no migration in or out of the population

Godfrey Hardy and Wilhelm Weinberg went on to develop a simple equation that can be used to discover the
probable genotype frequencies in a population and to track their changes from one generation to another. This
has become known as the Hardy-Weinberg equilibrium equation. In this equation (p² + 2pq + q² = 1), p is defined
as the frequency of the dominant allele and q as the frequency of the recessive allele for a trait controlled by a
pair of alleles (A and a). In other words, p equals all of the alleles in individuals who are homozygous dominant
(AA) and half of the alleles in people who are heterozygous (Aa) for this trait in a population. In mathematical
terms, this is p = AA + ½Aa.
Likewise, q equals all of the alleles in individuals who are homozygous recessive (aa) and the other half of the
alleles in people who are heterozygous (Aa). q = aa + ½Aa
Because there are only two alleles in this case, the frequency of one plus the frequency of the other must equal
100%, which is to say p + q = 1.
Since this is logically true, then the following must also be correct: p = 1 – q.
There were only a few short steps from this knowledge for Hardy and Weinberg to realize that the chances of all
possible combinations of alleles occurring randomly is
(p + q)² = 1 or more simply p² + 2pq + q² = 1
In this equation, p² is the predicted frequency of homozygous dominant (AA) people in a population, 2pq is the
predicted frequency of heterozygous (Aa) people, and q² is the predicted frequency of homozygous recessive (aa)
ones.
From observations of phenotypes, it is usually only possible to know the frequency of homozygous recessive
people, or q² in the equation, since they will not have the dominant trait. Those who express the trait in their
phenotype could be either homozygous dominant (p²) or heterozygous (2pq). The Hardy-Weinberg equation
allows us to predict which ones they are. Since p = 1 - q and q is known, it is possible to calculate p as well.
Knowing p and q, it is a simple matter to plug these values into the Hardy-Weinberg equation (p² + 2pq + q² = 1).
This then provides the predicted frequencies of all three genotypes for the selected trait within the population.

49. B. It is an example of aposematic coloration.


Many prey species blend in with their environment, making it difficult for the predators to find them. These species
use camouflage as their first line of defense. Another word for this type of defense is "crypsis" or "cryptic
coloration." Cryptic coloration is especially common in small animals such as insects, lizards, snakes, and frogs.
These animals are often the same color as the leaves or twigs on which they rest. Some insects even look like the
twigs or leaves themselves. It is important to remember that crypsis is not just a morphological adaptation, but
that behavior plays a very important part as well. Crypsis works only if the animal is resting on the appropriate
background and usually only when the animal isn't moving.

Many small animals have evolved toxic chemicals that make the creature poisonous to eat. Interestingly, many of
these species are brightly colored, making it easy for the predators to see them. Scientists believe that the bright
coloration has evolved to help the predator, often birds, remember that the species is poisonous. For example, if a
bird eats a poisonous butterfly or frog, it will get very sick. In some cases, the poison is released so quickly that
the bird will spit the prey out and avoid swallowing it. In either case, it is probably easier for the bird to remember
to avoid this species in the future if the prey is distinctively colored. Experiments have shown that it often takes
only a single encounter with a toxic prey species for a predator to learn to avoid it. Warning coloration,
sometimes referred to as aposematic coloration, is found in a wide variety of animals, including insects, mites,
spiders, and frogs.

50.B. One of the keys to understanding evolution lies in understanding the phenomenon of speciation, the
underlying cause of variation that results in the creation of new species.

Many neo-Darwinists have argued that the variation acted upon by natural selection leading to the origins of new
species is caused by mutation. In fact, the notion of mutation as the major driver of biodiversity has become the
prevailing wisdom.
Scientists have induced mutations in various species in the laboratory for years. Some of these induced mutations
have been passed along to offspring, demonstrating that mutations are indeed heritable. Yet no scientist has been
successful in creating a new species in the laboratory by inducing mutation in an existing species.
In addition to the lack of evidence supporting mutation as the cause of variation leading to speciation, more often
than not mutation is deleterious. Hermann J. Muller, the Nobel laureate, showed that X-rays are mutagenic in
fruit flies. He also pointed out that 99.9% of the mutations are negative and do not get selected for. So while
mutation clearly plays a role in evolution, and it may be a major reason for variation within a species, it is not the
driving force of speciation.
There is ample evidence supporting the hypothesis that speciation, and in turn biodiversity, stems largely from
genomic acquisition. The protagonist of this gigantic creative activity turns out to be a midget in stature – the
microbe. To borrow a phrase from Darwin, there is grandeur in this view.
One of the modern day champions of the theory of genomic acquisition is Lynn Margulis, a biologist at the
University of Massachusetts, Amherst, who against considerable opposition early in her career proposed and
defended the endosymbiont theory of mitochondria and chloroplasts, which is now subscribed to by many
biologists and supported by overwhelming evidence.

In organisms that reproduce sexually, most of the genetic variation in a population results from the unique
combination of alleles that each individual receives from its parents. Of course, at the nucleotide level, all the
differences among these alleles have originated from past mutations and other processes that can produce new
alleles. But it is the mechanism of sexual reproduction that shuffles existing alleles and deals them at random to
produce individual genotypes. Three mechanisms contribute to this shuffling: crossing over, independent
assortment of chromosomes, and fertilization. During meiosis, homologous chromosomes, one inherited from each
parent, trade some of their alleles by crossing over. These homologous chromosomes and the alleles they carry
are then distributed at random into gametes. Then, because myriad possible mating combinations exist in a
population, fertilization brings together gametes that are likely to have different genetic backgrounds.
The combined effects of these three mechanisms ensure that sexual reproduction rearranges existing alleles into
fresh combinations each generation, providing much of the genetic variation that makes evolution possible.

51. A. The Hardy-Weinberg equilibrium is a principle stating that the genetic variation in a population will
remain constant from one generation to the next in the absence of disturbing factors. When mating is random in a
large population with no disruptive circumstances, the law predicts that both genotype and allele frequencies will
remain constant because they are in equilibrium.
The Hardy-Weinberg equilibrium can be disturbed by a number of forces, including mutations, natural selection,
nonrandom mating, genetic drift, and gene flow. For instance, mutations disrupt the equilibrium of allele
frequencies by introducing new alleles into a population. Similarly, natural selection and nonrandom mating
disrupt the Hardy-Weinberg equilibrium because they result in changes in gene frequencies. This occurs because
certain alleles help or harm the reproductive success of the organisms that carry them. Another factor that can
upset this equilibrium is genetic drift, which occurs when allele frequencies grow higher or lower by chance and
typically takes place in small populations. Gene flow, which occurs when breeding between two populations
transfers new alleles into a population, can also alter the Hardy-Weinberg equilibrium.
If, in a hypothetical population of size N, the birth rates (per capita) are represented as b and death rates (per
capita) as d, then the increase or decrease in N during a time period t will be:

dN/dt=(b-d)*N

(b-d) is called the 'intrinsic rate of natural increase', o r value, and is a very important parameter chosen for
assessing the impacts of any biotic or abiotic factor on population growth.
Any species growing exponentially under unlimited resource conditions can reach enormous population densities
in a short time. Darwin showed how even a slow growing animal like the elephant could reach an enormous
population if there were unlimited resources for its growth in its habitat.
In order to quickly come to the answer, note that the higher the initial population and the higher the r value, the
higher increase in the coming year will be. Because D and E populations are very large, you can predict that they
will have greatest increases. You can quickly calculate that in D the increase will be 20 000 and in E it will be
30 000. So you can choose answer E.
If you have time, you can come back later to this question and calculate all numbers:
A=200 000 * 0.020=4 000
B=500 000 * 0.040=20 000
C=2 000 000 * 0.008=16 000.

IV is a molecule, therefore it should be the smallest (rule out answers B and E). Now A, C, and D differ in second
choice, but it is hard to compare those because you probably do not know the length of chromosome at metaphase
and the diameter of a secretory vesicle. Notice that they differ in the last choice as well, and you can guess that the
length of Euglena flagellum (which is visible to the naked eye) is greater than the diameter of nucleus (which is
hardly visible to the naked eye) .So you know that the order should be III->II. This leaves you with the choice D as
a correct one.
Human metaphase chromosomes have a chromatid width of about 0.6 μm, while the length of' an individual
chromosome varies from 1 μm (chromosome No. 20) to 5 μm (chromosome No. 1).
In mammalian cells, the average diameter of the nucleus is approximately 6 micrometres (µm).
Body is fusiform, rounded anteriorly, gradually tapering posteriorly forming a tip. Dimensions: 40 to 60 μm long
and 10 to 18 μm in diameter. Flagellum is of the same length as the body, or slightly longer.
The quickest amino acid must be the most positive so that it easily goes to positive pole (anode) and smallest so
that it easily squeezes through pores in the gel. The smallest amino acid in this case is glycine, however, it is
neutral. Leucine is large but does not have a charge. Lysine is large and has a positive charged which repel
positive anode and therefore will move slowest, or will be attracted to cathode (negative pole). Aspartic acid and
glutamic acid are both very large and negatively charged at neutral pH, thus they will move fastest. If you
compare their molecular weights, you will realize that aspartic acid is smaller and will move faster than glutamic
acid. Thus, the answer should be A.
The order in which I would put amino acids is below:
Cathode (+) Lysine -> leucine ->glycine->glutamic acid ->aspartic acid ->Anode (-)

pKa 1 represents the carboxyl group and pKa2 represents the protonated amine group.

When the pH is considerably lower than the pKa we expect both sides to be fully protonated.
There will be no charge at the carboxy, and a positive charge at the nitrogen for a net charge of +1.
When we raise the pH a few units above the first pKa, and still well below the second pKa value, the carboxyl group
will lose its proton; however, the amino group is still protonated. This is the zwitterion form, with a positive and
negative to cancel out.
When you raise the pH to well above the amino value, the nitrogen will lose its proton and thus its charge. We now
have negative and zero for a net charge of -1.

At pH 7 basic amino acids will be positively charged and so they will behave as cations. Acidic amino acids will be
negatively charged and so they will behave as anions. Neutral amino acids will be both positively and negatively
charged (zwitterions) and so they will stay in the centre. These amino acids are said to be at their isoelectric point.
Each type of amino acid has its isoelectric point at a particular pH, it depends upon the number of ionisable groups
in the molecule. At any pH above the isoelectric point the molecule will have a net negative charge and move
towards the anode. Similarly at pHs below the isoelectric point the molecule will have a net positive charge and
move towards the cathode.
Let’s calculate the number of moles of each molecule:
n(arachidonic acid)=m/Mw=10.0 g / 304 g/mol = 0.033 mol
n(iodine)= m/Mw=33.3 g / 254 g/mol = 0.131 mol
From the illustration above you can see that 2 mol of iodine are needed to react with 2 double bonds, thus 1 iodine
atom is needed to react with 1 double bond. The ratio is 1:1.
Let’s compare our ratio: 0.131 mol : 0.033 mol = 3.96 which roughly means 4.
The answer is D.

Statement A seems the most probable. Myoglobin is a protein molecule that has a similar structure and function to
hemoglobin. It is a smaller monomer of polypeptide structure, a globular protein with amino acids and prosthetic
heme group binds to proximal histidine group while a distal histidine group interact on the other side of the plane. It
binds and stores oxygen without concerning cooperativity. Almost all proteins have conformational changes that
occur all of the time and myoglobin is no different. In fact myoglobin and haemoglobin exhibit a ‘breathing’ motion
in that they expand and contract continuously so their structure opens and closes all of the time on a nanosecond
timescale. There are cavities in the protein structure so the oxygen would essentially diffuse through the protein
very rapidly via these cavities.

Interestingly, examination of a static structure of myoglobin, i. e. one obtained from the crystallographic studies,
shows no open pathways whatsoever. The structure is very much closed, with only a small number of tiny cavities
open, but disconnected. Then how can an oxygen molecule penetrate through to the heme group? To answer this
question, one has to consider the dynamical properties of myoglobin.
Myoglobin works at physiological temperatures of about 300 K at which it exhibits considerable thermal motion. It
also exists in a highly dynamic environment of the cellular cytoplasm. What permits oxygen into myoglobin are
fluctuations of its structure, inevitable under the thermal conditions under which myoglobin performs its function.
Due to the thermal fluctuations of the residues comprising the protein, there is a possibility for the structure to have
random cavities where oxygen molecules can fit in. The random cavities are spreading throughout the protein, and
O2 molecules can travel along with these openings. As a result, the interior of myoglobin, statically apparently
closed to oxygen, is dynamically actually open. However, due to the probabilistic nature of the fluctuations that
allow for oxygen migration inside myoglobin, it is not straightforward for O2 to reach the heme. An oxygen
molecule is moving through the randomly opening cavities until it accidentally approaches the heme, a process
which is very long (ns to µs) in comparison with the time scale of local movements of the protein residues (ps).
The random opening and migration of the small cavities over a course of long dynamics, arising due to the flexibility
of the protein, make it possible for O2 to explore a substantial part of myoglobin’s interior and to reach the heme.
That is why one does not talk about oxygen channels in myoglobin (there are no real structural channels); instead,
the term “pathway” is used.
Statement D does not seem to happen as it tends to remain buried to prevent it reacting with other functional groups
and requires specific amino acids to sit alongside it in order for it to reversibly bind oxygen. If this was true, this
would lead to oxidative damage of heme.
Statements B and E are illogical.
Statement C could be right, but the tertiary structure of myoglobin and haemoglobin are almost identical so this
wouldn’t be something specific to myoglobin function
65. B. The synthesis of thiamine diphosphate (ThDP), also known as thiamine pyrophosphate (TPP) or
cocarboxylase, is catalyzed by an enzyme called thiamine diphosphokinase according to the reaction thiamine +
ATP → ThDP + AMP. ThDP is a coenzyme for several enzymes that catalyze the transfer of two-carbon units and
in particular the dehydrogenation (decarboxylation and subsequent conjugation with coenzyme A) of 2-oxoacids
(alpha-keto acids). Examples include:
Present in most species
• pyruvate dehydrogenase and 2-oxoglutarate dehydrogenase (also called α-ketoglutarate dehydrogenase)
• branched-chain α-keto acid dehydrogenase
• transketolase
Present in some species:
• pyruvate decarboxylase (in yeast)

The enzymes transketolase, pyruvate dehydrogenase (PDH), and 2-oxoglutarate dehydrogenase (OGDH) are all
important in carbohydrate metabolism. The cytosolic enzyme transketolase is a key player in the pentose
phosphate pathway, a major route for the biosynthesis of the pentose sugars deoxyribose and ribose. The
mitochondrial PDH and OGDH are part of biochemical pathways that result in the generation of adenosine
triphosphate (ATP), which is a major form of energy for the cell. PDH links glycolysis to the citric acid cycle,
while the reaction catalyzed by OGDH is a rate-limiting step in the citric acid cycle.
Pyruvate dehydrogenase catalyzes oxidative decarboxylation of pyruvate, to form acetyl-CoA. Without acety-CoA
Krebs cycle cannot proceed.
66. B. Pantothenic acid, also called pantothenate or vitamin B5 (a B vitamin), is a water-soluble vitamin. For
many animals, pantothenic acid is an essential nutrient. Animals require pantothenic acid to synthesize coenzyme-
A (CoA), as well as to synthesize and metabolize proteins, carbohydrates, and fats.
Pantothenic acid is used in the synthesis of coenzyme A (CoA). Coenzyme A may act as an acyl group carrier to
form acetyl-CoA and other related compounds; this is a way to transport carbon atoms within the cell. CoA is
important in energy metabolism for pyruvate to enter the tricarboxylic acid cycle (TCA cycle) as acetyl-CoA, and
for α-ketoglutarate to be transformed to succinyl-CoA in the cycle. CoA is also important in the biosynthesis of
many important compounds such as fatty acids, cholesterol, and acetylcholine. CoA is incidentally also required
in the formation of ACP, which is also required for fatty acid synthesis in addition to CoA.

67. C. Most of the metabolic energy generated in living cells results from processes that abstract electrons from a
donor molecule, channel them through an electron transport chain, and finally deliver them to an acceptor
molecule.
When the donor is NADH and the acceptor is oxygen, the overall reaction is strongly exergonic and ATP can be
generated (oxidative phosphorylation).
Primary electron donors of respiratory electron transport in the mitochondria are either NADH 1 H 1 or
metabolites that can be oxidized by FAD-dependent dehydrogenases. NADH delivers reducing equivalents to
NADH:ubiquinone oxidoreductase (complex I), a large complex of at least 40 subunits. It transfers the electrons to
ubiquinone (coenzyme Q, UQ), and at the same time translocates protons from the matrix (N phase) to the
intermembrane space (P phase). Complex I contains flavin mononucleotide (FMN), more than six iron–sulfur
clusters and bound ubiquinones.
A second reaction feeding the UQ pool is catalysed by succinate dehydrogenase. This enzyme, actually a
component of the Krebs cycle, is an integral component of the inner mitochondrial membrane, and therefore
commonly classified as complex II of the chain. As in complex I, electron transfer involves a flavin (flavin–adenine
dinucleotide, FAD, in this case), several iron–sulfur clusters, and bound quinones. In addition, the enzyme
contains a type cytochrome of unknown function. Further electron donors for the reduction of UQ are metabolites
such as acyl–CoA, a-glycerophosphate or choline. In these cases, the electron transfer flavoprotein (ETF) serves
as an intermediate electron carrier between the respective dehydrogenase and ubiquinone.
Reduced ubiquinone (ubiquinol, UQH2) delivers two hydrogen atoms to ubiquinol:cytochrome c reductase
(complex III) where the electrons are channelled to the next mobile carrier, cytochrome c, while the protons are
translocated into the intermembrane space. This pathway, which is known as the ‘Q-cycle’, involves two separate
binding sites for UQ and/or UQH2 (Qo and Qi), both connected to the UQ pool. Two b-type cytochromes (bH and
bL), the ‘Rieske’ [2Fe–2S] iron–sulfur protein and a ctype cytochrome mediate the transfer of electrons from Qo
to Qi or to cytochrome c, respectively. The respiratory chain is completed by cytochrome c oxidase (complex IV),
which transfers electrons to the final acceptor O2, again pumping protons from the matrix side into the
intermembrane space. The electrons delivered by cytochrome c first pass a copper centre (CuA) and haem a
before they reach a binuclear CuB–haem a3 site, where molecular oxygen is ultimately reduced to water in a
stepwise fashion.
In the illustration below notice the presence of many iron atoms associated with various complexes.
Statement I is correct because in aerobic organisms, oxygen is used as the electron acceptor. The electron
transport chain in bacteria is located in the cell membrane. Many important biochemical reactions, such as
energy generation, use concentration gradients across membranes. The general lack of internal membranes in
bacteria means reactions such as electron transport occur across the cell membrane between the cytoplasm and
the periplasmic space.
Statement II is correct because the bacterium does not have mitochondria and does not use oxygen for lactic acid
fermentation. In anaerobic organisms other inorganic compounds, such as nitrate, sulfate or carbon dioxide are
used as electron acceptors. This leads to the ecologically important processes of denitrification, sulfate reduction,
and acetogenesis, respectively. Another way of life of chemotrophs in the absence of possible electron acceptors is
fermentation, wherein the electrons taken from the reduced substrates are transferred to oxidised intermediates to
generate reduced fermentation products (e.g., lactate, ethanol, hydrogen, butyric acid). Fermentation is possible,
because the energy content of the substrates is higher than that of the products, which allows the organisms to
synthesise ATP and drive their metabolism.
Statements III and IV should also be correct. During anaerobic respiration 2 ATP are produced and 2 NADH are
oxidized to NAD+. Because glycerol metabolism requires one molecule of ATP and one molecule of NAD+,
anaerobic pathway fulfills energy requirements by generating, in a redox-balanced manner, 1 mol of ATP per mol
of glycerol converted to ethanol/lactic acid/ or other compound. During aerobic respiration, more ATP and more
NADH are produced, thus, more ATP and NAD+ can be generated to allow glycerol metabolism under aerobic
conditions.
Thus, my answer would be E.
4x = 2000. X=5.48 which is almost 6.Thus you should have at least 5 or more bases in your restriction target.
Because restriction enzymes most often recognize palindromic sequences, my
answer would be A.
A palindromic sequence is a nucleic acid sequence (DNA or RNA) that is the
same whether read 5' (five-prime) to 3' (three prime) on one strand or 5' to 3' on the complementary strand with
which it forms a double helix.

B. rRNA.

Klinefelter syndrome or Klinefelter's syndrome (KS) also known as 47,XXY or XXY, is the set of symptoms that
result from two or more X chromosome in males. The primary feature is sterility. Often symptoms may be subtle
and many people do not realize they are affected. Sometimes symptoms are more prominent and may include
weaker muscles, greater height, poor coordination, less body hair, smaller genitals, breast growth, and less
interest in sex. Often it is only at puberty that these symptoms are noticed. Intelligence is usually normal; however,
reading difficulties and problems with speech are more common. Symptoms are typically more severe if three or
more X chromosomes are present.
The underlying mechanism involves at least one extra X chromosome in addition to a Y chromosome such that
there is a total of 47 or more chromosomes rather than usual 46. KS is diagnosed by the genetic test known as a
karyotype.

If nondisjunction occurred during meiosis I of oogenesis, then we can get gametes with 22+XX or 22+O.
When a normal sperm (22+X or 22+Y) fertilizes these gametes, we can get the following offsprings: 44+XXX
(triple X syndrome), 44+XXY (Kleinfelter syndrome), 44+XO (Turner syndrome), 44+YO (inviable).

If nondisjunction occurred during meiosis II of oogenesis, then we can get gametes with 22+X, 22+XX or 22+O.
When a normal sperm (22+X or 22+Y) fertilizes these gametes, we can get the following offsprings: 44+XXX
(triple X syndrome), 44+XXY (Kleinfelter syndrome), 44+XO (Turner syndrome), 44+YO (inviable), 44+XX
(normal), 44+XY (normal).

If nondisjunction occurred during meiosis I of spermatogenesis, then we can get gametes with 22+XY or 22+O.
When a normal oocyte (22+X) is fertilized by these gametes, we can get the following offsprings: 44+XXY
(Kleinfelter's sundrome), 44+XO (Turner syndrome).
If nondisjunction occurred during meiosis II of spermatogenesis, then we can get gametes with 22+XY, 22+O,
22+X, 22+Y.
When a normal oocyte (22+X) is fertilized by these gametes, we can get the following offsprings: 44+XXY
(Kleinfelter's syndrome), 44+XO (Turner syndrome), 44+XX (normal), 44+XY (normal).

Aberrant crossing over may result in some genes being absent or present I greater amount but not in the whole
chromosome missing or being extra.

Thus answers can be A, B, C, and D.

Remember that yeasts are unicellular fungi and fungi are eukaryotes. Fungi do not have chloroplasts and thus are
heterotrophs. The answer should be A.
Three classes of mutations were identified in Arabidopsis.

Class A: flowers with these mutations have (unfused) carpels instead of sepals in whorl 1, and stamens instead
of petals in whorl 2. The pattern of organs (from outside to inside) is carpel, stamen, stamen, carpel. The genes
containing these mutations were namedAPETALA1 (AP1) and APETALA2 (ap2). Below is an image of
an apetala2 mutant flower (right) next to a wild type flower (left).

wild type flower from apetala2 mutant flower

Class B: flowers with these mutations have sepals in whorl 2 instead of petals, and (unfused) carpels in whorl 3
instead of stamens. The pattern of organs (from outside to inside) is sepal, sepal, carpel, carpel. The genes
containing these mutations were named APETALA3 (AP3) and PISTILLATA (PI). Below is an image of
a pistillata mutant flower (right) next to a wild type flower (left).

wild type flower pistillata flower

Class C: flowers with this mutation have petals in whorl 3 instead of stamens, and sepals in whorl 4 instead of
carpels. In addition, the floral meristem is not determinate - flowers continue to form within the flowers, so the
pattern of organs (from outside to inside) is: sepal, petal, petal; sepal, petal, petal; sepal, petal, petal, etc. The
gene containing this mutation was called AGAMOUS (AG). Below is an image of an agamous mutant flower
(right) next to a wild type flower (left).

The ABC model:

Each class of genes is required in two adjacent whorls. Class A genes are required in whorls 1 and 2, class
B genes are required in whorls 2 and 3, and class C genes are required in whorls 3 and 4. Both class
A and class B genes are required in whorl 2, and both class B and class C genes are required in whorl 3.
The ABC model that summarizes these results is shown below. We'll talk in class about how this model has
been tested.
Molecular basis of differentiation:
The A, B, and C genes are transcription factors. Different transcription factors are needed together to turn on
a developmental gene program--such as A and B needed to initiate the program for petals.
What turns on the different transcription factors in different cells?
• Induction and inhibition by one cell signalling to a neighbouring cell.
• Repression of expression of A gene by C product, and of C gene by A product.
• Numerous other developmental gene products regulate or respond to A, B, and C.

The answer should be D because both petals and stamens will be lacking.

The correct answers are A, B, and E.


In some plants, the lateral bud located in the axil of each leaf does not grow to form branches, especially at first.
This condition is known as strong apical dominance.

Apical dominance is thought to be caused by the apical bud producing IAA (auxin) in abundance. This auxin is
transported basipetally from the apical bud. The auxin causes the lateral buds to remain dormant.
When the apical bud is removed, the source of IAA is removed. Since the auxin concentration is much lower, the
lateral buds can now grow. In fact their growth will be stimulated by a relatively small drop in auxin
concentration (see graph). Thus, decapitating (pruning) a shoot will cause it to branch!

Such response is the basis for all pruning "to form." By pruning carefully, the branching pattern and form of the
plant can be controlled. If pruning limits plant growth to two dimensions against a wall, we call it espalier. Three-
dimensional pruning to form would be a kind of sculpture, called topiary. Perhaps you have observed topiary in
the form of Mickey and Minnie Mouse at Disney World. If both shoot and root pruning is done frequently, the
result can be a bonsai, the creation of a miniature tree. Frequently bonsai trees are not only dwarf in stature, but
also respond to the severe pruning by producing dwarf leaves as well. Bonsai is an art form that involves
miniature landscaping around the dwarf tree.

Cytokinins are involved in many plant processes, including cell division and shoot and root morphogenesis. They
are known to regulate axillary bud growth and apical dominance. The "direct inhibition hypothesis" posits that
these effects result from the cytokinin to auxin ratio. This theory states that auxin from apical buds travels down
shoots to inhibit axillary bud growth. This promotes shoot growth, and restricts lateral branching. Cytokinin
moves from the roots into the shoots, eventually signalling lateral bud growth. Simple experiments support this
theory. When the apical bud is removed, the axillary buds are uninhibited, lateral growth increases, and plants
become bushier. Applying auxin to the cut stem again inhibits lateral dominance.

Statement A is incorrect because the walls are unevenly thickened:

Statement B is incorrect because the outer covering of the plant is cell wall, not epidermis. The dermal system
consists of a layer of tightly packed cells called the epidermis. On most plant stems and leaves, the epidermis is
covered with a waxy coating called the cuticle, which helps prevent water loss through the epidermis. The
epidermis is the protective outer layer of covering all plant organs.

Statement C is correct as well. A meristem is the tissue in most plants containing undifferentiated cells
(meristematic cells), found in zones of the plant where growth can take place.
Meristematic cells give rise to various organs of the plant and keep the plant growing. The shoot apical meristem
(SAM) gives rise to organs like the leaves and flowers, while the root apical meristem (RAM) provides the
meristematic cells for the future root growth. SAM and RAM cells divide rapidly and are considered
indeterminate, in that they do not possess any defined end status. In that sense, the meristematic cells are
frequently compared to the stem cells in animals, which have an analogous behaviour and function.

Statement D is correct:
Statement E is correct:

Statement A is incorrect because basement membrane is a characteristic of animal cells. The basement membrane
is a thin, fibrous, non-cellular region of tissue that separates the epithelium (skin, respiratory tract,
gastrointestinal tract, etc), mesothelium (pleural cavity, peritoneal cavity, pericardial cavity, etc) and endothelium
(blood vessels, lymph vessels, etc) from underlying connective tissue.
Statement B is incorrect because tissue fluid is found only in animal tissues.
Similarly, statement C is also incorrect because matrix is a characteristic of animal tissues, not plant tissues.
Because in the question you have got a tissue with a large number of fibers arranged in parallel bundles, you can
predict that it can be xylem. Because there are only two types of water-conducting cells, tracheids and vessel
Elements which are dead at functional maturity, the variety will be low (statement E is incorrect) whereas only
few alive cells should be present (statement D is correct).
You should notice that the plant exhibits Kranz anatomy. It is a special structure in the leaves of plants that have a
C4 pathway of carbon dioxide fixation. The leaves contain a ring of mesophyll cells, containing a few small
chloroplasts concerned with the initial fixing of carbon dioxide, surrounding a sheath of parenchyma cells (the
bundle sheath) which has large chloroplasts involved in the Calvin cycle.

Statement B is incorrect because C4 plants are adapted to hot, arid climates.


Statement C is incorrect because C4 plants firstly fix CO2 with PEP, not RuBP
Statement D is incorrect because carbon fixation, part of the Kalvin cycle, occurs in the stroma, not in the
thylakoid membranes.
Statement E is incorrect because the epidermal cells, except for guard cells, do not have chlorophylls.
Correct answer is A.
3 CO2 molecules are used for incorporation into 1 molecule of PGAL.
X CO2 molecules are used for incorporation into 8 x 10^6 molecules of PGAL
X= 24 x 10^6 molecules of CO2 consumed.

1 molecule of glucose – 6 molecules of CO2


2 x 10^6 molecules of glucose – Y molecules of CO2
Y=12 x 10^6 molecules of CO2 produced

Net molecules consumed = 24 x 10^6 - 12 x 10^6 = 12 x 10^6 molecules.


The answer should be D.

Oxygen is produced during water-splitting reaction from water, not from CO2, thus the answer should be D.

The answer should be E. The secondary walls of tracheids and vessel elements are hardened with lignin. This
hardening prevents collapse under the tensions of water transport and also provides support. Remember that the
main process that occurs in vessels and tracheids is water transport aided by transpiration, so you can rule out
statements A and D. Because transpiration is driven by negative pressure, i.e. drag rather than push (unlike
guttation which is caused by positive pressure), tracheids and vessel elements are under the risk of being pulled
inward, not outward.
The major histocompatibility complex (MHC) is a set of cell surface molecules encoded by a large gene family
which controls a major part of the immune system in all vertebrates. The major function of MHCs is to bind to
peptide fragments derived from pathogens and display them on the cell surface for recognition by the appropriate
T-cells. MHC molecules mediate interactions of leukocytes, also called white blood cells (WBCs), which are
immune cells, with other leukocytes or with body cells. The MHC determines compatibility of donors for organ
transplant, as well as one's susceptibility to an autoimmune disease via crossreacting immunization. In humans,
the MHC is also called the human leukocyte antigen (HLA).

MHC class II can be conditionally expressed by all cell types, but normally occurs only on professional antigen-
presenting cells (APCs): macrophages (statement D is correct; note that monocytes give rise to macrophages), B
cells, and especially dendritic cells (DCs). An APC takes up an antigen, performs antigen processing, and returns
a molecular fraction of it—a fraction termed the epitope—to the APC's surface, coupled within an MHC class II
molecule mediating antigen presentation by displaying this epitope. On the cell's surface, the epitope can contact
its cognate region on immunologic structures recognizing that epitope. That molecular region which binds to —
or, in jargon, ligates— the epitope is the paratope.

On surfaces of helper T cells are CD4 receptors (statement E is correct), as well as T cell receptors (TCRs). When
a naive helper T cell's CD4 molecule docks to an APC's MHC class II molecule, its TCR can meet and be
imprinted by the epitope coupled within the MHC class II. This event primes the naive helper T cell.

MHC class I occurs on all nucleated cells (statement A is correct) —in essence all cells but red blood cells—and
presents epitopes to killer T cells, also called cytotoxic T lymphocytes (CTLs). A CTL expresses CD8 receptors, in
addition to TCRs. When a CTL's CD8 receptor docks to a MHC class I molecule, if the CTL's TCR fits the epitope
within the MHC class I molecule, the CTL triggers the cell to undergo programmed cell death by apoptosis. Thus,
MHC class I helps mediate cellular immunity, a primary means to address intracellular pathogens, such as viruses
and some bacteria, including bacterial L forms, bacterial genus Mycoplasma, and bacterial genus Rickettsia.

Statement B is correct as well. All dendritic cells (DCs) have functional MHC class I and MHC class II
presentation pathways. MHC class I molecules present peptides that are derived from proteins degraded mainly in
the cytosol, which in most DC types comprise almost exclusively endogenous proteins (synthesized by the cell
itself). MHC class II molecules acquire peptide cargo that is generated by proteolytic degradation in endosomal
compartments. The precursor proteins of these peptides include exogenous material that is endocytosed from the
extracellular environment, and also endogenous components, such as plasma membrane proteins, components of
the endocytic pathway and cytosolic proteins that access the endosomes by autophagy. CD8+ DCs have a unique
ability to deliver exogenous antigens to the MHC class I (cross-presentation) pathway, although the mechanisms
involved in this pathway are still poorly understood.

Statement C is incorrect one. Memory B cells are also antigen-presenting cells which have MHCII as well as
CD4+ receptors.
The answer is C. III->I->V->VII->VI->IV->II.

The answer is B. Anterior pituitary secretes gonadotropins. Gonadotropes, primarily luteinising hormone (LH)
secreted from the anterior pituitary stimulates the ovulation cycle in female mammals, whilst in the males, LH
stimulates the synthesis of androgen which drives the ongoing will to mate together with a constant production of
sperm.
Gonadal hormones – nearly always synonymous with gonadal steroids – are hormones produced by the gonads,
and include both steroid and peptide hormones. The major steroid hormones include estradiol and progesterone
from the ovaries, and testosterone from the testes. Gonadal hormones generally exert their effects via nuclear
receptors, but can also work via membrane receptors such as GPCRs.
The answer is B. Sodium-potassium pumps move two potassium ions inside the cell as three sodium ions are
pumped out to maintain the negatively-charged membrane inside the cell; this helps maintain the resting potential.
Statement A is incorrect because acetylcholine is able to bind to nicotinic receptors and open sodium channels.
This leads to positively charged ions moving inside the cell and depolarizing it. If the threshold is reached, the
neuron will fire an action potential.
Statement C is incorrect because it is opposite what normally occurs in the cell. When the neuronal membrane is
at rest, the resting potential is negative due to the accumulation of more sodium ions outside the cell than
potassium ions inside the cell.
Statements D and E are incorrect, Potassium ions diffuse out of the cell at a much faster rate than sodium ions
diffuse into the cell because neurons have many more potassium leakage channels than sodium leakage channels.
The correct order should be A->C->E->D->B.
Thus the answer should be D.

The correct answer is E. Acetylcholinesterase, also known as AChE or acetylhydrolase, is a hydrolase that
hydrolyzes the neurotransmitter acetylcholine. AChE is found at mainly neuromuscular junctions and cholinergic
brain synapses, where its activity serves to terminate synaptic transmission. During neurotransmission, ACh is
released from the nerve into the synaptic cleft and binds to ACh receptors on the post-synaptic membrane,
relaying the signal from the nerve. AChE, also located on the post-synaptic membrane, terminates the signal
transmission by hydrolyzing ACh. The liberated choline is taken up again by the pre-synaptic nerve and ACh is
synthetized by combining with acetyl-CoA through the action of choline acetyltransferase. Without
acetylcholinesterase, Ach accumulates in the synaptic cleft and is not broken down. This means that postsynaptic
neurons are activated for a longer than usual period and thus neurons will fire action potentials in uncontrolled
fashion.

87. B. Cancellous bone, synonymous with trabecular bone or spongy bone, is one of two types of osseous tissue
that form bones. The other osseous tissue type is cortical bone also called compact bone. The primary anatomical
and functional unit of cancellous bone is the trabecula. It is commonly thought that the trabecula within the
cancellous bone is aligned towards the mechanical load distribution that a bone experience.
The mineral component is composed of hydroxyapatite, which is an insoluble salt of calcium and phosphorus.
About 65% of adult bone mass is hydroxyapatite. Bone also contains small amounts of magnesium, sodium, and
bicarbonate.

88. B. A tetanic contraction (also called tetanized state or tetanus) occurs when a motor unit has been maximally
stimulated by its motor neuron. This occurs when a muscle's motor unit is stimulated by multiple impulses at a
sufficiently high frequency. Each stimulus causes a twitch. If stimuli are delivered slowly enough, the tension in
the muscle will relax between successive twitches. If stimuli are delivered at high frequency the twitches will
overlap resulting in tetanic contraction. When tetanized, the contracting tension in the muscle remains constant in
a steady state. This is the maximal possible contraction.

Notice in the diagram below that all action potentials are identical in strength. It is the frequency of action
potentials that changes.

89. A.
Tropomyosin and troponin are two other proteins found in small quantities in muscle. They help regulate muscle
contraction.
Troponin is associated with the thin filaments and can bind to the actin molecules. There is usually one troponin
per 6-8 actin molecules. Tropomyosin is a long thin protein that extends between, and binds to, the troponin
molecules. When troponin is bound to actin, the tropomyosin is positioned so it prevents the myosin heads
from contacting actin, thus preventing contraction.
The Contraction Process
· Electrical signal arrives at the cell.
· Electrical signal causes Ca2+ release into the cell.
· Ca2+ binds to troponin.
· Troponin/Ca2+ no longer binds actin.
· Tropomyosin is no longer blocking the myosin binding sites on actin.
· Myosin binds actin.
· ATP is hydrolyzed supplying the energy for the myosin heads to move with respect to the rod portion, causing the
actin filaments to be drawn towards the center of the sarcomere.
· This process is repeated many times; with each movement of a myosin head, the thin filament pulled further
inward.
90. A. The information processing zone is also known as association zones. Association areas take up an
increasingly larger percentage of the cerebral cortex as brain size increases among different species. Association
areas function to produce a meaningful perceptual experience
of the world, enable us to interact effectively, and support
abstract thinking and language. The parietal, temporal, and
occipital lobes - all located in the posterior part of the cortex -
integrate sensory information and information stored in
memory. The frontal lobe or prefrontal association complex is
involved in planning actions and movement, as well as abstract
thought. Globally, the association areas are organized as
distributed networks. Each network connects areas distributed
across widely spaced regions of the cortex. Distinct networks
are positioned adjacent to one another yielding a complex
series of interwoven networks. The specific organization of the
association networks is debated with evidence for interactions,
hierarchical relationships, and competition between networks.
In humans, association networks are particularly important to
language function. In the past it was theorized that language
abilities are localized in the left hemisphere in areas 44/45, the
Broca's area, for language expression and area 22, the
Wernicke's area, for language reception. However, language is
no longer limited to easily identifiable areas. More recent research suggests that the processes of language
expression and reception occur in areas other than just those structures around the lateral sulcus, including the
frontal lobe, basal ganglia, cerebellum, pons, and the caudate nucleus.

91. B. It should be balance, not auditory input which is perceived by hair cells covered by tectorial membrane and
found in cochlea.
The cupula is a structure in the vestibular system, providing the sense of spatial orientation. The cupula is located
within the ampullae of each of the three semicircular canals.
92. B. Receptors are detectors which transmit information to the brain. Depending on which neurons are
activated, we perceive different information about coldness. The brain interprets physical energy from the outside
world as nerve signals and processes them into ways that they can be used.
A thermoreceptor is a sensory receptor, or more accurately the receptive portion of a sensory neuron, that codes
absolute and relative changes in temperature, primarily within the innocuous range. In the mammalian peripheral
nervous system, warmth receptors are thought to be unmyelinated C-fibres (low conduction velocity), while those
responding to cold have both C-fibers and thinly myelinated A delta fibers (faster conduction velocity). The
adequate stimulus for a warm receptor is warming, which results in an increase in their action potential discharge
rate. Cooling results in a decrease in warm receptor discharge rate. For cold receptors their firing rate increases
during cooling and decreases during warming. Some cold receptors also respond with a brief action potential
discharge to high temperatures, i.e. typically above 45°C, and this is known as a paradoxical response to heat.
The mechanism responsible for this behavior has not been determined. A special form of thermoreceptor is found
in some snakes, the viper pit organ and this specialized structure is sensitive to energy in the infrared part of the
spectrum.

In humans, temperature sensation enters the spinal cord along the axons of Lissauer's tract that synapse on
second order neurons in grey matter of the dorsal horn, one or two vertebral levels up. The axons of these second
order neurons then decussate, joining the spinothalamic tract as they ascend to neurons in the ventral
posterolateral nucleus of the thalamus.

Cold-sensitive thermoreceptors give rise to the sensations of cooling, cold and freshness.

This area of research has recently received considerable attention with the identification and cloning of the
Transient Receptor Potential (TRP) family of proteins. The transduction of temperature in cold receptors is
mediated in part by the TRPM8 channel. This channel passes a mixed inward cationic (predominantly carried by
Na+ ions although the channel is also permeable to Ca2+) current of a magnitude that is inversely proportional
to temperature. The channel is sensitive over a temperature range spanning about 10-35°C. TRPM8 can also be
activated by the binding of an extracellular ligand. Menthol can activate the TRPM8 channel in this way. Since the
TRPM8 is expressed in neurons whose physiological role is to signal cooling, menthol applied to various bodily
surfaces evokes a sensation of cooling. The feeling of freshness associated with the activation of cold receptors by
menthol, particularly those in facial areas with axons in the trigeminal (V) nerve, accounts for its use in numerous
toiletries including toothpaste, shaving lotions, facial creams and the like.

Another molecular component of cold transduction is the temperature dependence of so-called leak channels
which pass an outward current carried by potassium ions. Some leak channels derive from the family of two-pore
(2P) domain potassium channels. Amongst the various members of the 2P-domain channels, some close quite
promptly at temperatures less than about 28°C (e.g. TRAAK, TREK). Temperature also modulates the activity of
the Na+/K+-ATPase. The Na+/K+-ATPase is a P-type pump that extrudes 3Na+ ions in exchange for 2K+ ions
for each hydrolytic cleavage of ATP. This results in a net movement of positive charge out of the cell, i.e. a
hyperpolarizing current. The magnitude of this current is proportional to the rate of pump activity.

I (air) -> III (ear ossicles: malleus, incus, stapes) -> II. liquid -> perilymph which through basal membrane cause
vibrations of endolymph.
Thus, the answer should be B.

B. Color vision is the ability of an organism or machine to distinguish objects based on the wavelengths (or
frequencies) of the light they reflect, emit, or transmit. The basis for difference in the ability to see color lies in the
number of cone types that differ between species. Mammals in general have color vision of a limited type, and
usually have red-green color blindness, with only two types of cones. Humans, some primates, and some
marsupials see an extended range of colors, but only by comparison with other mammals. Most non-mammalian
vertebrate species distinguish different colors at least as well as humans, and many species of birds, fish, reptiles
and amphibians, and some invertebrates, have more than three cone types and probably superior color vision to
humans.
C. Gonadotropins are secreted from pituitary gland, not ovary and it targets ovary, not uterus. The two principal
gonadotropins in vertebrates are luteinizing hormone (LH) and follicle-stimulating hormone (FSH).
Gonadotropins are released under the control of gonadotropin-releasing hormone (GnRH) from the arcuate
nucleus and preoptic area of the hypothalamus. The gonads — testes and ovaries — are the primary target organs
for LH and FSH. The gonadotropins affect multiple cell types and elicit multiple responses from the target organs.
As a simplified generalization, LH stimulates the Leydig cells of the testes and the theca cells of the ovaries to
produce testosterone (and indirectly estradiol), whereas FSH stimulates the spermatogenic tissue of the testes and
the granulosa cells of ovarian follicles, as well as stimulating production of estrogen by the ovaries.

A formula called "Hamilton's Rule" looks like this:


rB>C, where (in terms of fitness) c indicates the cost to the actor, b
indicates the benefit to the recipient, and r indicates the degree of
relatedness between actor and recipient. By this formulation, actors
can be expected to gain fitness by assisting relatives, provided that
the cumulative benefit to the recipients is greater than the cost to
the actor.
r between grandmother and you is ¼, between you and uncle is ¼
and between you and your first cousin is 1/8.
Let’s assume that each has a chance to produce X additional
offsprings.
So, if C is the likelihood of your survival during rescuing, without
your help Hamilton’s rule would look like:
B= X (the number of your own offsprings) + X * (1/4 + ¼ + 1/8)
*0.25=1.15625X.
With your help the equation would look like this:
B=C*X + X * (1/4 + ¼ + 1/8) *0.75= X*0.625*0.75=0.46875X +
CX
0.46875X + CX > 1.15625*X
0.46875 + C > 1.15625
C > 0.6875
C > 68.75 %. This means that survival must be greater than 68.75% or risk of drowning lesser than 100-
68.75%=31,25 %.
The answer should be C.

Statement A is unlikely because if A-chick is larger than B-chick, B chick has less nutrients and is more susceptible
to environmental hazards.
Statement B is quite persuasive, however, the word ‘equivalent’ here sounds misleadingly. Remember that eggs
have a hatching failure rate of 25-40% which means that in one-egg clutch only roughly 60-75% of offsprings will
survive whereas in two-egg clutch survival rate can be 60-75% if the first egg survives and then pushes other egg
out of the nest, 0.25x0.6= 15% or a bit higher if the first dies and the second survives.
Statement E is also unlikely because in the problem we cannot find any information about environmental
circumstances apart from the fact that if an egg is pushed from the nest, exposed egg has a higher rate of dying.
Now C and D look similar. If the A-egg fails to hatch (in 25-40% of cases), there is 60-75% rate of hatching of a
second egg. Thus, this statement is possible. If a hatched A-organism dies within its first few days, it is incapable
of pushing out the second egg, thus this can be raised to maturity. Therefore, D is also correct.
My answers are C and D.

The correct answer is D. Stars. Indigo Buntings migrate at night, using the stars for guidance. Researchers
demonstrated this process in the late 1960s by studying captive Indigo Buntings in a planetarium and then under
the natural night sky. The birds possess an internal clock that enables them to continually adjust their angle of
orientation to a star—even as that star moves through the night sky.
Statement C is incorrect because birds do not perceive odors.

ssDNA means that the organism has single-stranded genetic material. Because the DNA is not in double-stranded
form, we cannot use complementarity rules to predict the percentages, thus the answer should be E.

The question is incorrect; the question needed to specify that the parents of the water bender were both water
benders - if you make that assumption the answer is 33%. But let’s solve this problem without corrections.
So we have three alleles:
IW> IA> i
An air bender male can be IAIA or IAi. As we are told that he came from a true-breeding lineage, meaning that his
parents were homozygous, he also should be homozygous, thus his genotype is IAIA.
A water bender female can be IW IW, IW IA, IW i. Her both parents are benders but a brother is nonbender (ii), thus
we can predict that both parents were heterozygotes and each had one recessive i allele: _i x _i where I can be
either IW or IA (IW i x IWi, IWi x IAi, IAi x IW i). If her parents were IW i x IWi, there is 1/3 chance that she is IW IW and
2/3 chances she is IWi. If her parents were IWi x IAi or IAi x IW i, then the probability that she is IWIA is ½ and IWi –
½ (because we know she is water not air bender and she is bender and not nonbender).
We have three types of grandparents. So a chance of each of the cross to get the daughter is 1/3. Let’s calculate
the overall probability for a daughter to be water bender:

1/3 (If her parents were IW i x IWi) * 1/3 (she is IW IW)=1/9


1/3 (If her parents were IW i x IWi) * 2/3 ( she is IWi) = 2/9
2/3 (If her parents were IWi x IAi or IAi x IW i) * ½ (she is IWIA ) = 1/3
2/3 (If her parents were IWi x IAi or IAi x IW i) * ½ (she is IWi )=1/3

So the father will pass on to his child IA allele with 100% chance. Now you need to consider what chances are for
a mother to pass IW allele.
When male and female mate, we can get the following children which are bender:
1) If the woman is IW IW we will have no children air benders.
2) If the woman is IWi (p=2/9), then ½ of her children will be air benders. P = ½ * 2/9 =1/9
3) If the woman is IWIA (p=1/3), then½ of her children will be air benders. P = ½ * 1/3 =1/6
4) If the woman is IWi (p=1/3), then ½ of her children will be air benders. P = ½ * 1/3 =1/6.
The overall probability is thus 1/9 + 1/6 + 1/6 = 0.44 = 44%.

A – probability of survival P(A) = P(A/B)*P(B)+ P(A/Bc)*P(Bc)


B – probability of C-section 0.98 = 0.96*0.15 + P(A/Bc)*0.85
Bc- probability of no C-section 0.98-0.144= P(A/Bc)*0.85
A/B – probability of surviving when C-section is done 0.836= P(A/Bc)*0.85
In our case we need to find P(A/ Bc). P(A/Bc)=98.35%
p(A) = 0.98. P(B) = 0.15, P(Bc)=0.85, P(A/B)=0.96. The answer should be C.

Statement A is correct because this would explain the development of the peripheral nervous system.
Statement B is correct because
cephalization is one of the major trends in
evolution:
Statement C is correct because
interneurons, or association neurons,
make up the largest part of the CNS.
Statement E is correct as well because if
you consider the eyes of Platyhelminthes
and humans you can notice the increased
complexity in the latter species.
Statement D is incorrect because all
neurons carry impulses in only one
direction: from dendrites to axons.
Two major trends characterize the evolution of nervous systems in the bilaterally symmetrical phyla of animals:
centralization and cephalization. Centralization of nervous systems refers to a structural organization in which
integrating neurons are collected into central integrating areas rather than being randomly dispersed.
Cephalization is the concentration of nervous structures and functions at one end of the body, in the head. Both
trends can be seen even in flatworms, which belong to the phylum Platyhelminthes, considered the most ancient
phylum to have bilateral symmetry. Apparently the presence of a distinct anterior end and the development of a
preferred direction of locomotion in bilateral animals have been important in the evolution of centralized,
cephalized nervous systems.

In flatworms and animals of more complex bilaterally symmetrical phyla, centralization is anatomically evident by
the presence of longitudinal nerve cords, discrete aggregations of neurons into longitudinally arranged clusters
and tracts to constitute a distinct CNS. Motor neurons extend out from the CNS to effectors, and sensory neurons
extend from the periphery of the body into the CNS. Increasing numbers of interneurons—neurons that are neither
sensory nor motor and are confined to the CNS—make their appearance as nervous systems become more
complex. The interneurons enhance capacities for centralized integrative processing in the nervous system. The
peripheral nervous system (PNS) also is increasingly consolidated in bilaterally symmetrical animals. Instead of a
random meshwork of processes running in all directions in an unpolarized nerve net, the peripheral sensory and
motor processes are coalesced into nerves, discrete bundles of nerve axons running between the CNS and the
periphery.

Cephalization, the other general evolutionary trend in nervous system organization, involves varying degrees of
anterior concentration of nervous system organization. In the most primitive of centralized nervous systems, each
region of the CNS largely controls just its own zone or segment of the body (see Parts 3 and 4 of the figure);
indeed, elements of such segmental or regional organization persist in all phyla, including vertebrate chordates.
In most bilaterally symmetrical animals, however, the most anterior part of the CNS exerts a considerable degree
of domination and control over other regions. This anterior part, typically larger and containing more neurons
than other parts, is called the brain. Brain is a general term for an anterior enlargement of the CNS.

Cephalization is thought to be an evolutionary adaptation resulting from the tendency of bilaterally symmetrical
animals to move forward, so that information about newly encountered parts of the environment impinges first on
the front of the animal. As a correlate of forward motion, most groups with bilateral symmetry have evolved
anterior placement of many of their major sense organs. The anterior placement of the brain allows it to receive
environmental information from these sense organs with a minimum of neural connection lengths and
corresponding delays. The relative importance of the anterior brain then led to varying degrees of its dominance
over the rest of the CNS.

Among vertebrates, a trend somewhat analogous to cephalization occurs within the brain: The relative size of the
forebrain (anterior part of the brain) increases successively in reptiles, nonprimate mammals, and primates.
Along with this development, functions formerly controlled by the spinal cord or brainstem come increasingly
under forebrain control.
103. E.
By definition, polyploidy just means that a cell or organism contains more than 2 pairs of homologous
chromosomes (or is more than 2n). This is more common in plants than it is in animals. The plant, as shown
below, undergoes failed meiosis, which means that the diploid (2n) cells never
become haploid (n). As a result, a plant ends up with more than 2n when it
self-pollinates. The shown result is tetraploidy (4n), but there are other
possible results (3n, 5n, etc).
Multiple plants within a population can end up with the same polyploidy
number. They can then reproduce with each other but not with the original
plants or any other plants. As a result, they become biologically isolated from
the original group of plants and are considered a different species. It is a type of sympatric speciation, which
means that it occurs without geographic isolation.

104. B. Genetic drift describes random fluctuations in the numbers of gene variants in a population. Genetic drift
takes place when the occurrence of variant forms of a gene, called alleles, increases and decreases by chance over
time. These variations in the presence of
alleles are measured as changes in allele
frequencies.
Typically, genetic drift occurs in small
populations, where infrequently occurring
alleles face a greater chance of being lost.
Once it begins, genetic drift will continue until
the involved allele is either lost by a
population or until it is the only allele present
in a population at a particular locus. Both
possibilities decrease the genetic diversity of a
population. Genetic drift is common after
population bottlenecks, which are events that
drastically decrease the size of a population.
In these cases, genetic drift can result in the
loss of rare alleles and decrease the gene
pool. Genetic drift can cause a new population to be genetically distinct from its original population, which has
led to the hypothesis that genetic drift plays a role in the evolution of new species.

105. D. Because biological species are defined in terms of reproductive compatibility, the formation of a new
species hinges on reproductive isolation—the existence of biological factors (barriers) that impede members of
two species from interbreeding and producing viable, fertile offspring. Such barriers block gene flow between the
species and limit the formation of hybrids, offspring that result from an interspecific mating.
In the problem we have postzygotic barrier described because a hybrid does not develop into a fertile adult. This
is termed as reduced hybrid fertility. Even if hybrids are vigorous, they may be sterile. If the chromosomes of the
two parent species differ in number or structure, meiosis in the hybrids may fail to produce normal gametes. Since
the infertile hybrids cannot produce offspring when they mate with either parent species, genes cannot flow freely
between the species.

106. A and C. In allopatric speciation, a physical barrier


splits a single ancestral population into two or more
populations that are no longer able to exchange genes.
Because gene flow between the populations is prevented,
each population acts as an independent evolutionary
lineage. Different mutations will arise in each population
and different substitutions will also occur due to random
effects of genetic drift and local differences in natural
selection. Thus, given a sufficient time period, genetic
differences will accumulate between the two physically
separated populations. Presumably, this genetic
divergence is accompanied by the evolution of
reproductive isolation. When the nascent species come
back into physical contact (i.e. the barrier is removed),
reproductive isolating mechanisms prevent gene
exchange and thus, preserve species differences.

Least related species will be the one with the highest number of differences.
Number of differences between:
A – 0.
B – 1.
C – 1.
D – 2.
E – 1.
Thus, the answer should be D.
Here we will use chi square.
If the hypothesis was true, then we can have 4 types of crosses between parent species:
1)Aa x Aa
2)AA x AA
3)Aa x AA
4)AA x Aa
Because we got a second phenotype, only cross 1) can account for a different phenotype than parental.
Our hypothesis is that normal wings are dominant over dumpy wings and there is no difference between expected
and observed values.
In total we have 120 offsprings, of which 83 have normal wings and 37 have dumpy wings. If we perform cross 1),
the offsprings should be in 3:1 phenotypic ratio.
Now we can use chi-square to confirm or deny our hypothesis. Here is the equation that we will use:

Let’s quickly make a table:


Observed Expected (O-E)2
Normal wings 83 120*3/4=90 49
Dumpy wings 37 120*1/4=30 49

X2=49/90 + 49/30 = 0.544 + 1.633 = 2.177.


In our case we have two phenotypic classes, thus degrees of freedom are Df=N-1=2-1=1.
Now go back to the table above and notice that in order to reject our hypothesis with 95% probability and 1
degree of freedom we need to get chi value greater than 3.84. Our value is less (2.177 < 3.84) so we accept our
hypothesis and conclude that normal wings are dominant over dumpy wings. The answer should be B.
If you quickly make a graph:

# survivors
1200

1000
Number of survivors

800

600

400 # survivors

200

0
0 10 50 70 90 100
Percentage of maximum lifespan

You can conclude that the species has Type I survivorship curve. Survivorship curves show how mortality varies
with age of the individuals of the cohort. Age-specific mortality, as well as age-specific fecundity, is due to
changing susceptibilities and capabilities of the individual, and the variation in its environmental exposure.
Depending on the age at which most of the mortality takes place, organisms can have different survivorship
curves. Deevey (1947) described three types of curves (Fig. 1): Type I occurs when survival of young is high, and
mortality increases drastically towards the end of the lifespan. This is typical of very protected life styles. In case
of Type II survivorship curves, mortality is constant with age, as in a decay process. Most organisms have a Type
III curve, where most individuals die when they are young, while older
individuals are good survivors.
The correct statements are II, III, and V so you should choose answer E.

K-selected, or equilibrium, species are species whose population sizes tend


to be limited by carrying capacity (K). These organisms tend to be large in
size and provide greater care for their relatively few offspring. Their
population sizes vary within a relatively narrow range above and below
their environment's carrying capacity. The pressures on K -selected species
tend to be density dependent, meaning that as the population density goes
up, environmental resistance will assert itself more strongly.
r-selected or opportunist species are species whose population sizes tend to be limited by reproductive rate. They
are often small in size and provide little or no care to their numerous offspring. Their population sizes fluctuate
greatly in response to changes in their environment. The pressures on r -selected species tend to be density
independent and generally come from physical forces such as temperature and rain that are unrelated to the
population density.
Three idealized survivorship curves describe when species members are likely to die within the species' life span:
late loss (type I), constant loss (type II), and early loss (type III).

110. A. Gause’s competitive exclusion principle states that no two species can coexist if they occupy the same
niche. In other words, an ecological niche cannot be simultaneously and completely occupied by stabilized
populations of more than one species. In his classic experiment Gause first grew Paramecium caudatum and
Paramecium Ha in separate cultures and found that each species grew in numbers according to the logistic
equation. However, P. aurelia grows in numbers more quickly than P. caudatum and shows more individuals in
the same volume of culture medium.

But when he grew the two species together in same culture volume, he observed that initially both species grew in
numbers, but eventually P. caudatum declined and became extinct. He repeated has experiment and found that P.
aurelia always won the competition between the two species. Gause attributed the result to the nee of ‘but a single
niche in the conditions of the experiment’.
111.A. The competitive exclusion principle is an ecological principle stating that when two competing life forms
attempt to occupy the same niche, only one outcome is possible: One life form will drive out the other. If any
members of the other remain, it is only because they have adapted, and are now living in a slightly different niche.

112. B. Organisms that exhibit boom-and-bust growth curve are likely to be r-strategists which have Type III
survivorship curves. Usually, their mortality is density-independent, therefore, not crowding but environmental
factors most likely determine the population size in this insect species.

1. R-stretegy
R-strategies principally operate to maximize
reproductive rates in harsh, unstable environment or
during habitat disturbance, while there is only a little
competition. R-strategies favor a “gambling”
strategies. Some characters for r-strategist listed in
table below.

2. K-strategy
Principally, these individuals tend to maintain their
populations at a more or less constant level, close to
the carrying capacity of environment. They prefer not
to increase a number of young, but they apply their
high survival ability. Some characters for K-strategist listed in table below.
R-Strategist K-Strategist
Body size small Body size large
Short generation time Long generation time
High level of dispersal Low level of dispersal
The mortality is density- The mortality is density
independent dependent
High reproductive rate and Low reproductive rate and
often reach overshoot rarely reach overshoot
Population density Population density relative
fluctuative “boom and constant from generation to
bust” generation
Tend to be poor Good competitors or keen
competitors
Good colonization ability The population only contains a
few of individuals
No food spezialitation Usually they restrict just to few
of plant
They lives in They usually lives in a stable
unpredictable environment environment

Rarely able to persist High survival rate, especially


successfully for long reproductive stages
period in certain habitat
Able to quickly discover
new habitats or nomadens
Examples: grasshoppers Examples : Morabine
(Chortoicetes grasshoppers (Geckomima sp.)
terminifera), wasp scelio
fulgidus, Bushfly Musca
vestustissima,

A birth rate or death rate that does not change with population density is said to be density independent. In a
classic study of population regulation, Andrew Watkinson found that the mortality of dune fescue grass (Vulpia
membranacea) is mainly due to physical factors that kill similar proportions of a local population, regardless of
its density. For example, drought stress that arises when the roots of the grass are uncovered by shifting sands is a
density-independent factor. In contrast, a death rate that rises as population density rises is said to be density
dependent, as is a birth rate that falls with rising density. Watkinson and Harper found that reproduction by dune
fescue declines as population density increases, in part because water or nutrients become more scarce. Thus, the
key factors regulating birth rate in this population are density dependent, while death rate is largely regulated by
density-independent factors.
Density-dependent regulation provides that feedback, halting population growth through mechanisms that reduce
birth rates or increase death rates. Main factors are:
• Competition for Resources
• Predation
• Toxic wastes
• Intrinsic physiological factors
• Territoriality
• Disease

113. A. In some species that produce many offspring but provide little care for them (r-selected species), mortality
is greatest among the youngest individuals. The Type III survivorship curve indicative of this life history is initially
very steep, which is reflective of very high mortality among the young, but flattens out as those individuals who
reach maturity survive for a relatively longer time; it is exhibited by animals such as many insects or shellfish.
114. A. The statement again describes r-strategists which mortality is density-independent. Density-independent
factors, typically physical or chemical in nature (abiotic), influence the mortality of a population regardless of its
density. They include weather, natural disasters, and pollution. An individual deer may be killed in a forest fire
regardless of how many deer happen to be in that area. Its chances of survival are the same whether the
population density is high or low.

Here you need to use a rule of 10%: An average of only


10% of the energy in one trophic level is passed to the
next trophic level. Three main reasons explain it:
• Energy is “used up” for daily life activities.
• Not all organisms that die are eaten by animals in
the next trophic level.
• Not all parts of an organism are eaten and
digested for energy.

Because herbivores are primary consumers, they should


have 10% less energy: 20 000 * 0.1 = 2000 kcal. The
answer is D.

The answer is B. Notice that statement B and C are opposite to each other so you can consider only those two
statements.
The answer is D which is a predator with highest food pyramid level. Remember that at each step in the food
chain, the amount of these toxic chemicals increases.
Bioaccumulation refers to the accumulation of substances, such as pesticides, or other chemicals in an organism.
Bioaccumulation occurs when an organism absorbs a toxic substance at a rate greater than that at which the
substance is lost. Thus, the longer the biological half-life of
the substance the greater the risk of chronic poisoning, even if
environmental levels of the toxin are not very high.
The level at which a substance accumulates depends on the
rate of uptake (through the gills of a fish, contact with the
skin). How quickly the substance is removed depends on the
metabolic process, the lipid content of the organism and other
environmental and physical factors. The more hydrophobic a
substance is the more likely it is to bioaccumulate. The
pesticide is passed on to other organisms when the ones with
pesticides contained in the fatty tissues are eaten
(biomagnification). The concentration of the pesticide is then
increased as a predator will eat numerous of the prey.

The answer is B. The subphylum Chelicerata constitutes one of the major subdivisions of the phylum Arthropoda.
It contains the horseshoe crabs, sea spiders, and arachnids (including
scorpions and spiders).
Like all arthropods, chelicerates have segmented bodies with jointed limbs,
all covered in a cuticle made of chitin and proteins. The chelicerate bauplan
consists of two tagmata, the prosoma and the opisthosoma, except that mites
have lost a visible division between these sections. The chelicerae, which
give the group its name, are the only appendages that appear before the
mouth. In most sub-groups, they are modest pincers used to feed. However,
spiders' chelicerae form fangs that most species use to inject venom into
prey. The group has the open circulatory system typical of arthropods, in
which a tube-like heart pumps blood through the hemocoel, which is the
major body cavity. Marine chelicerates have gills, while the air-breathing
forms generally have both book lungs and tracheae. In general the ganglia
of living chelicerates' central nervous systems fuse into large masses in the
cephalothorax, but there are wide variations and this fusion is very limited
in the Mesothelae, which are regarded as the oldest and most primitive
group of spiders. Most chelicerates rely on modified bristles for touch and for information about vibrations, air
currents, and chemical changes in their environment. The most active hunting spiders also have very acute
eyesight.
The answer is C.
Phytoplankton biomass is usually measured by the amount of chlorophyll-a in the water. Chlorophyll-a is a
photosynthetic pigment that serves as a measurable parameter for all phytoplanktonic production. On average,
1.5% of algal organic matter is chlorophyll-a. Thus, if chlorophyll-a levels are known, one can estimate the
phytoplankton biomass in the water body. High biomass will discolor the water body.
Six taxonomic divisions delineate the different types of algae: Chrysophyta, Pyrrophyta, Phaeophyta, Rhodophyta,
Chlorophyta and Euglenophyta.

1. Division Chrysophyta: This division contains 6650 unicellular species, including golden algae,
yellow-green algae, and diatoms.
o Class Chrysophyceae (golden algae): Most members are flagellated and are found in
both fresh and salt water. The class contains 500 species
o Class Xanthophyceae (yellow-green algae): Most are non-motile and are found in
fresh, brackish, and salt water. The class contains 550 species.
o Class Bacillariophyceae (diatoms): Members do not have a flagella and many are non-
motile. Most occur as plankton and are found in both fresh and salt water. The diatoms
have a characteristic thin siliceous shell (cell wall). The shells contain minute
depressions which form intricate patterns that are used to identify different species. The
class contains 5600 living species.

2. Division Pyrrophyta (dinoflagellates): Most members of this division are unicellular


biflagellates. Most occur as plankton and are found in both fresh and salt water. The
dinoflagellates have thick cellulose plates that look like armor. This division has 1000 species.
Blooms of dinoflagellates may pose health and environmental risks. These blooms are known as "red tides"
because they color the sea red or brown. Dinoflagellates secrete a poison that can be toxic when there are many
organisms in a small area, such as occurs during blooms. Many aquatic organisms, especially those in the
benthos, may perish.

3. Division Phaeophyta (brown algae): Brown algae is a multicellular algae that grows
primarily in salt water. Sizes range from microscopic to the largest existing seaweed (kelp).
The division contains 1500 species.
4. Division Rhodophyta (red algae): Red algae is a multicellular algae of which 3900 species
occur in salt water and 100 species occur in fresh water. Red algae is usually found attached to
a substrate.
5. Division Chlorophyta (green algae): The green algae are very diverse. Most green algae
occur in fresh water, although a few groups are marine. Some green algae is found in snow,
soil, on tree trunks and in symbiosis with other organisms. This division contains 7000 known
species that are either unicellular or multicellular. Division Chlorophyta is separated into
three major classes: Charophyceae, Ulvophyceae, and Chlorophyceae.
o Class Charophyceae: Algae of this class are either unicellular, few- celled, or
filamentous in nature. Spirogyra, a slimy filamentous algae, is a well-known resident of
fresh water impoundments.
o Class Ulvophyceae: This flagellated-cell algae occurs only in marine environments.
o Class Chlorophyceae: Most green algae belong to this class. Except for a few
planktonic marine groups, Chlorophyceae primarily occurs in fresh water (Raven et al.,
1986).

6. Division Euglenophyta (euglenoids): All euglenoids but one, Colacium, are unicellular. Most
euglenoids are freshwater organisms. Sizes range from less than 10 micrometers in length to
more than 500 micrometers. Division Euglenophyta contains more than 800 species.

Alfalfa, also called lucerne, is a perennial flowering plant in the pea family Fabaceae cultivated as an important
forage crop in many countries around the world. Thus, it belongs to eudicots.
Lilium (members of which are true lilies) is a genus of herbaceous flowering plants growing from bulbs, all with
large prominent flowers. They belong to monocot family Liliaceae.
Marigolds are annual or perennial, mostly herbaceous plants in the sunflower family (Asteraceae or Compositae),
thus they belong to eudicots.
Rice, Oryza sativa, commonly known as Asian rice, belongs to monocot family Poaceae.
The tomato is the edible, often red fruit/berry of the nightshade Solanum lycopersicum, commonly known as a
tomato plant. The tomato belongs to the eudicot nightshade family, Solanaceae.
Monocot stems are usually herbaceous and include plants such as lilies, corn, rice and other grasses; eudicot
stems may be herbaceous or woody. Examples of herbaceous eudicots include tomatoes, marigolds, alfalfa,
pansies and petunias and examples of woody eudicots include pecan trees, oaks and maples.
Thus, the answer should be A, C, and E.
Characteristic Annelids Arthropods
Segmentation Present. The fundamental characteristic of Present. Body of primitive forms is
the phylum is the division of the body into a homonomous (all body segments are very
linear series of cylindrical segments, or similar), but in vast majority there is some
metameres. Each metamere consists of a degree of tagmatization (specialization and
section of the body wall and a compartment of fusion of joints).
the body cavity with its internal organs.
Appendages Movement involves coordinating longitudinal 3, 4 or 5 pairs of legs, some have wings.
and circular muscles in each segment with Jointed appendages are modified for walking,
the fluid-filled coelom functioning as a feeding, sensory reception,
hydrostatic skeleton. copulation and defence.
• Circular muscle contraction makes each
segment thinner and longer;
longitudinal muscle contraction makes the
segment shorter and thicker.
• Waves of alternating contractions pass
down the body.
• Most aquatic annelids are bottom-dwellers
that burrow, although some swim in
pursuit of food.
Nervous system The annelid nervous system is composed of a Their nervous system is "ladder-like", with
pair of cerebral ganglia lying above and paired ventral nerve cords running through
anterior to the pharynx. all segments and forming paired ganglia in
• A nerve ring around the pharynx connects each segment. Their heads are formed by
these ganglia to a subpharyngeal fusion of varying numbers of segments, and
ganglion, from which a pair of fused nerve their brains are formed by fusion of the
cords run posteriorly. ganglia of these segments and encircle the
• Along the ventral nerve cords are fused oesophagus.
segmental ganglia
Digestive The complete digestive system is divided intoThe digestive tube of arthropods is complete,
system several parts, each specialized for a specific
containing mouth and anus. Arthropods are
function in digestion: pharynx esophagus protostome animals, i.e., in their embryonic
crop gizzard  intestine. development the blastopore originates the
mouth.
Most arthropods have three main regions:
foregut, midgut and hindgut. The order that
food passes through the grasshopper
includes: the mouth, the oesophagus, which
conveys food from the pharynx to the crop, a
stomach of sorts. Next to the true stomach,
then the intestine, which leads to the rectum,
then to the anus.
Gas exchange Some aquatic annelids have thin-walled, • Feathery gills in aquatic species
feathery gills through which gases are • Tracheal systems in insects
exchanged between the blood and the • Book lungs in other terrestrial forms (e.g.,
environment. However, most annelids have no spiders)
special organs for gas exchange, and
respiration occurs directly through the body
wall.
Circulatory Closed circulatory system Open circulatory system
system

1.
a. Because the girl had a stunted growth, most probably she did not have enough somatotropin, a growth hormone
secreted from the pituitary gland. Therefore, this organ might have been affected.

b. Growth hormone, also called somatotropin, is a polypeptide hormone which stimulates growth and cell
reproduction.

c. Growth hormone deficiency has a variety of different negative effects at different ages; for example, in newborn
infants, the primary manifestations may be hypoglycemia or micropenis, while in later infancy and childhood,
growth failure is more likely. Deficiency in adults is rare, but may feature diminished lean body mass, poor bone
density, and a number of physical and psychological symptoms. Psychological symptoms include poor memory,
social withdrawal, and depression, while physical symptoms may include loss of strength, stamina, and
musculature. Other hormonal or glandular disorders frequently coincide with diminished growth hormone
production.

Severe prenatal deficiency of GH, as occurs in congenital hypopituitarism, has little effect on fetal growth.
However, prenatal and congenital deficiency can reduce the size of a male's penis, especially when gonadotropins
are also deficient. Besides micropenis in males, additional consequences of severe deficiency in the first days of
life can include hypoglycemia and exaggerated jaundice (both direct and indirect hyperbilirubinemia).

Even congenital GH deficiency does not usually impair length growth until after the first few months of life. From
late in the first year until midteens, poor growth and/or shortness is the hallmark of childhood GH deficiency.
Growth is not as severely affected in GH deficiency as in untreated hypothyroidism, but growth at about half the
usual velocity for age is typical. It tends to be accompanied by delayed physical maturation so that bone
maturation and puberty may be several years delayed. When severe GH deficiency is present from birth and never
treated, adult heights can be as short as 48-65 inches (122–165 cm).

Severe GH deficiency in early childhood also results in slower muscular development, so that gross motor
milestones such as standing, walking, and jumping may be delayed. Body composition (i.e., the relative amounts of
bone, muscle, and fat) is affected in many children with severe deficiency, so that mild to moderate chubbiness is
common (though GH deficiency alone rarely causes severe obesity). Some severely GH-deficient children have
recognizable, cherubic facial features characterized by maxillary hypoplasia and forehead prominence (said to
resemble a kewpie doll).

d. GH deficiency is treated by replacing GH with daily injections under the skin or into muscle. Until 1985,
growth hormone for treatment was obtained by extraction from human pituitary glands collected at autopsy. Since
1985, recombinant human growth hormone (rHGH) is a recombinant form of human GH produced by genetically
engineered bacteria, manufactured by recombinant DNA technology. In both children and adults, costs of
treatment in terms of money, effort, and the impact on day-to-day life, are substantial.

You might also like